You are on page 1of 59

EYE LIDS / LACRIMAL UHS Solved MCQ’s Rana

Haseeb

MCQs
EYE
Answers from Recommended Books
• Renu Jogi
• Parsons
• Kanski

1
UHS Solved MCQ’s Rana EYE
Haseeb

PREFACE
All praises go to Almighty ALLAH who is the one, who is the most Merciful, who gave me
potential and ability to make material contribution. Darood O Salam for the Holy Prophet
(PBUH) for giving me the inspiration and all the Holy figures for guiding me throughout
my life.
It is not how much knowledge/appreciation you give but how much love you put into it
matters more. I updated this edition for my juniors specially Batch 2016-2021. In this
book, I have added a lot of UHS past M.C.Q’s. The main thing is all M.C.Q’s are added topic
wise in respective subjects and references from recommended books are also given. This
is best book to practice your knowledge. It will be helpful in UHS exams for 4th year MBBS
students. Alhamdulillah now this book is very concise, to the point and devoid of
unnecessary details.
Valuable suggestions and healthy criticism aimed at improving this book will be
welcomed. Nobody is perfect except ALLAH Almighty. All the Best!

Rana Haseeb Nasir


https://www.facebook.com/mHaseebRajput98
E-mail: Ranahaseeb98@yahoo.com
Cell: 0342-6597196

2
EYE LIDS / LACRIMAL UHS Solved MCQ’s Rana
Haseeb

SECTION 04
EYE
Topic – 01 –– EYE LIDS .............................................................. 04
Topic – 02 –– LACRIMAL SYSTEM ........................................... 07
Topic – 03 –– CONJUNCTIVA ..................................................... 08
Topic – 04 –– CORNEA ............................................................... 12
Topic – 05 –– SCLERA ................................................................ 16
Topic – 06 –– LENS / CATARACT ............................................. 18
Topic – 07 –– GLAUCOMA.......................................................... 24
Topic – 08 –– NEURO-OPHTHALMOLOGY………………….…...29
(Retina, Optic Pathway, Optic Nerve & Nerve Palsies’) ..
Topic – 09 –– ORBIT / ANATOMY / OCULAR TRAUMA ............ 42
Topic – 10 –– SQUINT / ERRORS OF REFRACTION ................ 50
Topic – 11 –– CHOROID / IRIS / VITREOUS............................... 56

3
UHS Solved MCQ’s Rana EYE
Haseeb

TOPIC #

EYE LIDS
1
1. Ectropion can be treated by all of the d) None of the above
following procedures EXCEPT
Ans. (B) Ref. Renu Jogi as UHS exam
a) V-Y operation
6. Exophthalmos associated with thyroid
b) Skin muscle resection ophthalmopathy have following
c) Kuhnt-Szymanowski operation clinical signs except:
d) Skin graft a) Proptosis
Ans. (B) Ref. Kanski as UHS exam b) Ptosis
2. Distichiasis is: c) Lid retraction
a) Absent eyelashes d) Conjunctival chemosis
b) Accessory row of lashes e) Extraocular muscle thickening
c) Long eyelashes Ans. (B)
d) Misdirected eye lashes 7. A sixty years old farmer presents with
e) Thick eyelashes the complaint of slow growing nodule
with ulcerated center, for the last five
Ans. (B) as UHS exam years, at the temporal side of his right
3. Treatment of trichiasis includes lower lid. It is itching and sometimes
a) Epilation have a trivial bleeding. Its edges are
b) Electrolysis rolled and hand.There is no local
lymphadenopathy. What is your
c) Skin muscle resection
provisional diagnosis?
d) All of the above
a) Squamous cell Carcinoma
Ans. (D)
b) Basal cell carcinoma
4. Surgery of choice in cases where c) Molluscum contagiosum
multiple ptosis operations have failed
d) Solar burn in the wart
and levator action is poor
e) Sebaceous carcinoma
a) Fascia lata sling surgery
b) Skin muscle resection Ans. (B) Ref. Kanski as UHS exam
c) Levator resection 8. Blaskovics operation is done for
d) Fasanella-Servat operation a) Proptosis
Ans. (A) as UHS exam b) Ptosis
c) Lagophthalmos
5. Internal hordeolum is an acute
suppurative inflammation of d) Entropion
a) Zeis’s gland Ans. (B) as UHS exam
b) Meibomian gland 9. Levator palpebrae superioris muscle is
c) Moll’s gland supplied by

4
EYE LIDS / LACRIMAL UHS Solved MCQ’s Rana
Haseeb

a) 3rd nerve submandibular lymph nodes are not


b) 4th nerve palpable.The most likely diagnosis is:
c) 5th nerve a) Basal Cell Carcinoma
d) 7th nerve b) Kaposi Sarcoma
c) Meride Cell Tumor
Ans. (A) as UHS exam
d) Sebaceous Gland Carcinoma
10. A young lady with blepharitis presents
with a swelling on the right upper lid e) Squamous Cell Carcinoma
for the last one month; the commonest Ans. (A) as UHS exam
painless lid swelling is:
14. The term ‘madarosis’ means
a) Chalazion
a) Absence of eyelashes
b) Cyst of mol
b) Extra row of eyelashes
c) Cyst of zeis
c) Misdirected eyelashes
d) Internal hordeolum
d) None of the above
e) Stye
Ans. (A) Ref. Renu Jogi as UHS exam
Ans. (A) Ref. Renu Jogi as UHS exam
15. Paralytic ptosis is due to
11. The common causes of cicatricial a) Complete or partial 3rd nerve palsy
entropion include
b) 4th nerve palsy
a) Trachoma stage IV c) 6th nerve palsy
b) Ulcerative blepharitis d) 7th nerve palsy
c) Burns Ans. (A) Ref. Kanski as UHS exam
d) All of the above 16. A 25 years old male comes to the clinic
Ans. (D) Ref. Kanski as UHS exam with a 3 months history of painless
swelling on his right upper lid. On
12. A young lady of 20 presents with examination there is posterior
chronic irritation, burning, grittiness blepharitis and a smooth rounded
and photophobia in both eyes. On slit nodule slightly away from the lid
lamp examination the eye lashes are margin. What is your most probable
matted together with yellow crusts. diagnosis?
One of these crusts was removed with
forceps and the area started bleeding. a) Chalazion.
What is the most likely diagnosis? b) Dermoid
a) Amyloidosis of the lids c) Haemangioma
b) Preseptal Cellulitis d) Papilloma
c) Posterior Blepharitis e) Sebaceous cyst
d) Seborrheic Blepharitis Ans. (A)
e) Ulcerative Blepharitis 17. Sling surgery should be avoided in
Ans. (E) Ref. Parsons as UHS exam cases of ptosis with
13. A 70 years old lady has an ulcerated a) Very poor levator action
nodular mass on her right medial b) Poor Bell’s phenomenon
lower eyelid for the last 4 years. The c) Weak Muller’s muscle
central ulceration has raised rolled
d) Multiple failed surgery
edges with dilated blood vessels over
the lateral margins. Preauricular and Ans. (E) Ref. Parsons as UHS exam

5
UHS Solved MCQ’s Rana EYE
Haseeb

18. Chalazion is a chronic inflammatory a) Actinic Keratosis


granuloma of b) Basal cell carcinoma
a) Meibomian gland c) Malignant melanoma
b) Zeis gland d) Sebaceous cell carcinoma
c) Moll’s sweat gland e) Squamous cell carcinoma
d) Wolfring gland
Ans. (D) Ref. Kanski as UHS exam
Ans. (A) 24. Pseudoptosis is seen in
19. Tylosis is a) Phthisis bulbi
a) Hypertrophy and thickening of eyelid b) Enophthalmos
margin c) Both of the above
b) Inversion of eyelid d) None of the above
c) Senile eversion of eyelid Ans. (C) Ref. Parsons as UHS exam
d) Distortion of cilia 25. The commonest painless lid swelling
is:
Ans. (A)
a) Chalazion
20. Sling surgery should be avoided in
cases of ptosis with b) Cyst of moll
a) Very poor levator action c) Cyst of zeis
b) Poor Bell’s phenomenon d) Externalhordiolum
c) Weak Muller’s muscle e) Internalhordeolum
d) Multiple failed surgery Ans. (A) Ref. Renu Jogi as UHS exam
Ans. (B) as UHS exam
21. The term ankyloblepharon means
a) Absence of lid
b) Adhesion of lid margin
c) Narrow palpebral aperture
d) Adhesion of lid to globe
Ans. (B)
22. In Old age the commonest cause of
lower lid entropion is:
a) Lower lid laxity
b) Spasm of orbicularis oculi muscle
c) Conjunctival fibrosis
d) Decrease in the number of
eyelashes
e) Dry eyes
Ans. (A)
23. A recurrent chalazion should he
subjected to histopathologic
evaluation to exclude the possibility
of:

6
CONJUNCTIVA / CORNEA UHS Solved MCQ’s Rana
Haseeb

TOPIC #

2 LACRIMAL SYSTEM

1. The sequela of chronic dacryocystitis c) Meibomian gland


includes d) Salivary gland
a) Lacrimal abscess
Ans. (C)
b) Atonic sac
6. The accessory lacrimal glands are
c) Panophthalmitis
a) Glands of Krause
d) All of the above
b) Glands of Wolfring
Ans. (D) Ref. Renu Jogi as UHS exam
c) Both
2. Dacryocystorhinostomy is done for: d) None
a) Cataract
Ans. (C) Ref. Parsons as UHS exam
b) Ectropion
8. Nasolacrimal duct opens in:
c) Chronic dacryocystitis
a) Inferior meatus
d) Acute dactyoscnitis
b) Inferior turbinate
e) Acute orbital cellulitis
c) Middle meatus
Ans. (C) d) Middle turbinate
3. The tear film has e) Superior meatus
a) Mucous layer
Ans. (A) Ref. Parsons as UHS exam
b) Aqueous layer
9. Epiphora occurs in
c) Lipid layer
a) Iritis
d) All of the above
b) Trachoma
Ans. (D) Ref. Parsons as UHS exam c) Chronic dacryocystitis
4. In Dacryocystorhinostomy (DCR) d) Acute congestive glaucoma
operation, the window is made in the
lateral nasal wall is, at the level of: Ans. (C)
a) Ethmidal sinus 10. A patient of 45 years old presented with
b) Middle meatus facial palsy. The epiphora in this
patient was due to:
c) Superior meatus
a) Ectropion
d) Inferior meatus
b) Entropion
e) Lacrimal fossa
c) Hyper secretion of tears
Ans. (E) d) Lacrimal pump failure
5. All the following are serous acinous e) Lagophthalmos
glands EXCEPT
Ans. (D)
a) Lacrimal gland
b) Glands of Krause

7
TOPIC #

CONJUNCTIVA
3
1. SAFE strategy is recommended for the e) Tranta's dots
control of
Ans. (E)
a) Cataract
5. 8 years old child brought by his
b) Conjunctivitis parents with chief complaints of
c) Diabetic retinopathy mucopurulent discharge, redness and
d) Glaucoma swelling of both eyes for the last two
e) Trachoma months. On examination there are
follicles on the upper tarsal
Ans. (E) conjunctiva and superior corneal
2. Sub conjunctival hemorrhage is a pannus. Which one is the most
typical feature of: probable diagnosis?
a) Adenoviral conjunctivitis a) Allergic conjunctivitis
b) Corneal ulcer b) Chronic bacterial conjunctivitis.
c) Fungal Keratitis c) Ligneous Conjunctivitis
d) Ophthalmia neonatorum d) Trachoma
e) Subarachnoid hemorrhage e) Viral conjunctivitis

Ans. (A) Ans. (D) Ref. Kanski as UHS exam

3. A three years old child is presented 6. WHO grading of trachoma includes all
with bilateral subconjuctival except:
haemorrghes. He has been suffering a) TF follicles
from fever, severe cough and chest b) TI inflammation
infection for the last few days. The
c) TS scarring
commonest cause of subconjunctival
haemorrghe in this case can be: d) TT trichiasi
a) Fever e) TP pannus
b) Side effect of antibiotics Ans. (E) Ref. Renu Jogi as UHS exam
c) Cough 7. Follicles are commonly seen in
d) Chest infection conjunctivitis due to:
e) Trauma to the eye a) Alkali burns
Ans. (C) b) Seasonal allergy
c) Chalamydial infection
4. Following is the features of Trachoma
Except: d) Pseudomonous conjunctivitis
a) Corneal pannus e) Gonococcal conjunctivitis
b) Follicles Ans. (B)
c) Herbert's pit 8. Mucin layer tear film deficiency occurs
d) Papillae in:

8
CONJUNCTIVA / CORNEA UHS Solved MCQ’s Rana
Haseeb

a) Canalicular block c) Eczematous conjunctivitis


b) Herpetic keratitis d) Trachoma
c) Keratoconjunctivitis sicca Ans. (A) as UHS exam
d) Lacrimal gland removal 14. A 7 years old child presents in every
e) Dacyocystitis spring season for the last 4 years with
Ans. (C) Ref. Renu Jogi as UHS exam itching, redness, watering. There is no
purulent discharge. He has got a
9. A house officer was asked about the history of eczema. On examination
causes of a dilated pupil. The most there are no follicles but papillae both
common cause of a dilated pupil is: upper tarsal conjunctivae. There are
a) Trauma no preauricular lymph nodes. What is
b) Third cranial nerve palsy the diagnosis?
c) Pharmacological dilatation a) Adenoviral conjunctivitis.
d) Acute angle closure glaucoma b) Bacterial conjunctivitis.
e) Tonic pupil c) Chemical conjunctivitis
d) Trachoma
Ans. (C) as UHS exam
e) Vernal conjunctivitis
10. Natural protective mechanisms of
conjunctiva include Ans. (E) Ref. Kanski as UHS exam
a) Low temperature 15. Herbert’s pits are seen on the
b) Flushing due to tears a) Lid margin
c) Blinking of eyelids b) Palpebral conjunctiva
d) All of the above
c) Arlt’s line
Ans. (D) Ref. Renu Jogi as UHS exam d) Limbus
11. Eyes should not be bandaged in
Ans. (D) as UHS exam
a) Corneal ulcer
16. A patient presented with photophobia
b) Purulent conjunctivitis and watering in left eye. On
c) Glaucoma examination with fluorescein staining
d) Retinal detachment revealed dendritic ulcer which is
caused by:
Ans. (B) Ref. Renu Jogi as UHS exam
a) Fungus
12. The following ocular investigation has
b) Herpes zoster virus
a risk of anaphylactic shock:
c) Herpes simplex virus
a) B/Scan
d) Mycobacterium
b) Biometry
e) Staphylococci
c) FFA
d) HRT Ans. (C) Ref. Renu Jogi as UHS exam
e) OCT 17. As a complication of acute
mucopurulent conjunctivitis, the
Ans. (C) Ref. Kanski as UHS exam corneal ulcers that develop are
13. Cobblestone appearance of the a) Marginal
conjunctiva is seen in b) Central
a) Spring catarrh c) Anywhere on cornea
b) Angular conjunctivitis d) No where

9
Ans. (A) as UHS exam b) Staphylococci
18. Phlyctenular conjunctivitis is due to a) c) Streptococci
Pneumococcus d) Neisseria meningitidis
b) Pseudomonas pyocyanea Ans. (A) as UHS exam
c) Allergy to endogenous protein
24. Trantas nodules are seen in
d) Allergy to exogenous protein
a) Blepharoconjunctivitis
Ans. (C) Ref. Kanski as UHS exam
b) Vernal conjunctivitis
19. A young boy presented in emergency
with watering and photophobia in right c) Phlyctenular conjunctivitis
eye after a trauma, which test is most d) Herpes keratitis
appropriate? Ans. (A) Ref. Kanski as UHS exam
a) Applanation tonometry
25. Deficiency of vitamin A can cause all
b) Fluorescein staining
EXCEPT
c) Rose Bengal staining
d) Schirmer's test a) Xerosis
c) Tear break-up time b) Keratomalacia
Ans. (B) c) Night blindness
d) Dermoid
20. An infant presented with sticky
discharge both eyes and extreme Ans. (D)
congestion of conjunctiva. Provisional 26. Bitot’s spots are associated with
diagnosis is the ophthalmia
neonatorum which is caused by: a) Vitamin A deficiency
a) Diphtheria b) Vitamin D deficiency
b) E Coll c) Vitamin E dificiency
c) Gonococcus d) All of the above
d) Staph.aureous
Ans. (A) Ref. Kanski as UHS exam
e) Streptococcus
27. A 15 year old boy presents with
Ans. (C) progressive decrease in vision. He
21. Sequelae of trachoma include suffers from Vernal Kerato
a) Pseudoptosis Conjunctivitis. He was started using
some eye drops which have
b) Cicatricial entropion dramatically reduced itching and
c) Trichiasis photophobia some 2 years ago. He is
d) All of the above found to have Bilateral Posterior Sub
Capsular Cataracts with Visual
Ans. (D)
acuities of 6/12 both eyes. The most
22. Pinguecula is due to the infiltration of likely topical anti allergic drug to
a) Hyaline cause cataract is:
b) Lipid a) Cromoglycate
b) Dexamethasone
c) Calcium
c) Emedastine
d) Fatty acids
d) Ketrolac
Ans. (A) Ref. Renu Jogi as UHS exam e) Lodoxamide
23. Organism causing ophthalmia Ans. (B) Ref. Renu Jogi as UHS exam
neonatorum is
28. Bilateral fat-like nodular area on nasal
a) Neisseria gonorrhoeae side is described as

10
CONJUNCTIVA / CORNEA UHS Solved MCQ’s Rana
Haseeb

a) Pinguecula
b) Pterygium
c) Phlycten
d) Pemphigoid
Ans. (A)
29. A 30 year old male from Swabi has
returned from a business/pleasure trip
to Thailand and developed sore red
right eye. He went to the local
Ophthalmologist who prescribed
Chloramphenical Eye Drops but his
condition did not improve. On
examination his visual acuities are 6/9
both eyes. On Slit Lamp examination
he has minimal discharge with
bilateral conjunctival follicles and
epithelial infiltrates in both corneas.
Rest of the examination of the eyes is
normal. He has enlarged Pre auricular
lymph nodes and has been
complaining of urethral discharge.The
likely diagnosis is:
a) Allergic conjunctivitis.
b) Bacterial Conjunctivitis
c) Chlamydial Conjunctivitis.
d) Vernal Conjunctivitis
e) Rosacea Conjunctivitis
Ans. (C)
30. Promising treatment of epidemic
keratoconjunctivitis is by
a) Oxytetracycline
b) Sulphacetamide 30%
c) Chloramphenicol
d) Adenine arabinoside
Ans. (D) as UHS exam

11
TOPIC #

CORNEA
4
1. A consultant ophthalmologist b) Epithelium
during ward round ask his house c) Endothelium
officer regarding what is the normal
d) Bowman’s membrane
corneal hydration depends upon:
a) Epithelium Ans. (B) Ref. Renu Jogi as UHS exam
b) Bowman's membrane 6. Keratoconus has got association
c) Stroma with the following conjunctival
condition
d) Endothelium
a) Membranous conjunctivitis
e) Descemets membrane
b) Follicular conjunctivitis
Ans. (D) Ref. Kanski as UHS exam c) Subconjunctival haemorrhage
2. Treatment of impending perforation d) Vernal kerato conjunctivitis
of corneal ulcer includes all EXCEPT
e) Chemical burns
a) Contact lens
Ans. (D)
b) Acetazolamide (diamox)
c) Therapeutic corneal graft 7. The earliest symptom to occur in
corneal ulcer is
d) Cautery
a) Pain
Ans. (A) Ref. Parsons as UHS exam b) Photophobia
3. Central corneal ulceration may be c) Loss of sensation
associated with
d) Diminished vision
a) Herpes virus
Ans. (B) as UHS exam
b) Bacteria
c) Fungus 8. The most common organism
responsible for hypopyon corneal
d) All of the above ulcer is
Ans. (D) a) Staphylococcus
4. The best suture material for repair of b) Pneumococcus
corneal injuries is: c) Pseudomonas
a) 10/0 Nylon d) Candida albicans
b) 6/0 vicryl
Ans. (B) Ref. Parsons as UHS exam
c) 5/0 Ethiband
9. The diagnostic finding of fungal
d) 6/0 prolene keratitis is:
e) 8/0 virgin silk a) Satellite Lesions
Ans. (A) Ref. Kanski as UHS exam b) Hypopyon
5. Bullous keratopathy involves c) Epithelial defect
a) Descemet’s membrane d) Radial Keratitis

12
CONJUNCTIVA / CORNEA UHS Solved MCQ’s Rana
Haseeb

e) Subepithelial infilteration 14. Common cause of non-healing


corneal ulcer
Ans. (A)
a) Chronic dacryocystitis
10. Rupture of Descemet’s membrane is b) Raised intraocular pressure
seen in c) Diabetes mellitus
a) Keratoconus d) All of the above
b) Rubella
Ans. (D) as UHS exam
c) Glaucoma
d) Retinoblastoma 15. A 40 year old lady from Darra Adam
Khel presents with intense pain,
Ans. (A) Ref. Renu Jogi as UHS exam photophobia and decreased vision
11. The ocular sign of the Wilson's in both eyes. Her Visual Acuities are
disease is; 6/24 both eyes and she has Keratic
a) Droplet cataract Precipitates, cells in the anterior
b) Fleischer's Line chamber and posterior synechiae.
Intraocular pressures are 10 and 12
c) Haab's Striae
Right and left Eyes respectively. She
d) Kayser Fleischer's Ring also complains of lower back ache.
e) Stocker Line All of the below are appropriate
Ans. (D) treatments except:
a) Analgesics
12. A man presented with pain, redness,
b) Sub Tenon Steroids
decreased vision in the right eye. He
was diagnosed, as a case of Corneal c) Topical Moxifloxacin
Ulcer with Hypopion, which d) Topical Cycloplegics
medication should not be used; e) Topical Steroids
a) Analgesic Ans. (C)
b) Antibiotics
16. Band-shaped keratopathy is due to
c) Antifungal
a) Calcareous degeneration
d) Cyclo-plegics
b) Hyaline degeneration
e) Steroids
c) Fatty degeneration
Ans. (E) Ref. Parsons as UHS exam d) Elastotic degeneration
13. A farmer presented with a fleshy Ans. (B) Ref. Parsons as UHS exam
triangular conjunctival mass 17. In lamellar keratoplasty:
encroaching upon the nasal side of
the cornea for 3mm. The most a) Full thickness corneal graft is used
appropriate treatment will be: b) A combination of corneal and
a) Artificial Tears/decongestant conjuctival graft is used
b) Complete excision with c) Partial thickness corneal graft is
conjunctival auto-graft used
c) Excision biopsy d) Donor cornea is stitched on top of
recipient cornea
d) Excision with Mitomycin - C
application e) Hard contact lens is applied after
removing corneal epithelium of the
e) Topical Antibiotic / steroids recipient
combination
Ans. (C)
Ans. (D)

13
18. The commonest complication of anterior chamber. One of the
excessive use of steroids in the eye following is the first mandatory
is: procedure in the management of this
a) Keratoglobus patient?
b) Herpes simplex keratitis a) Anterior Chamber Paracentesis.
c) Trachoma b) Corneal Scrape for Microscopy
and Gram Staining.
d) Hyphaema
c) Vitreous tap
e) Exophthalmos
d) Topical antibiotics
Ans. (B) as UHS exam
e) Topical steroids
19. In non healing sloughing corneal
ulcer, where the perforation of Ans. (B)
cornea is apprehended, the best 23. A female of 40 years presents with
option is: painless decrease of vision for four
a) Scrapping of the corneal ulcer years. The patient has myopia with
irregular astigmatism. There is
b) Subconjuctival antibiotics
thinning of the central cornea. The
c) Evisceration most sensitive investigation for this
d) Conjuctival flap patient is:
e) Carholization of the ulcer bed a) Refractions
Ans. (D) Ref. Kanski as UHS exam b) Keratometary
20. Dendritic ulcer is caused by: c) Corneal topography
a) Bacteria d) Ocular biometry
b) Fungus e) Anterior segment OCT
c) Radiation Ans. (C) Ref. Kanski as UHS exam
d) Trauma 24. A 24 year old man presents with
e) Virus watering, photophobia and dimness
of vision in his right eye. On
Ans. (E) examination his visual acuity in the
21. Munson’s sign is seen in affected eye is 6/18 and normal in the
other eye. On fluorescein staining
a) Episcleritis
cornea shows a lesion with
b) Chalcosis branching pattern. His corneal
c) Keratoconus sensitivity is decrease d. What is
d) Retinal detachment most likely diagnosis?
a) Adenoviral keratoconjunctivitis
Ans. (C) as UHS exam
b) Autoimmune keratitis
22. A 45 year old farmer is cutting wheat c) Bacterial keratitis
when he feels something went into
d) Fungal keratitis
his right Eye 2 days afterwards his
right eye becomes painful and e) Herpes Simplex Keratitis (HSK)
photophobic and he is referred to a Ans. (E) Ref. Renu Jogi as UHS exam
tertiary care hospital. His visual 25. A 15 years old girl presents with
acuity in the right eye is 6/24 painless and gradual dimness of
improving to 6/12 with pin hole. The
vision in both eyes since last 4
right pupil is constricted but there is
years. She has a history of vernal
no RAPD. The eye is injected with
keratoconjunctivitis since the age of
signs of Keratitis with a few cells in 5 and frequent change in glasses for

14
CONJUNCTIVA / CORNEA UHS Solved MCQ’s Rana
Haseeb

four years. On examination her d) Cyanoacrylate adhesive


visual acuity is counting finger in e) Descemet Stripping Automated
both eyes and cornea looks clear but Endothelial Keratoplasty
bulging and conical. The most
probable diagnosis is? Ans. (E) Ref. Parsons as UHS exam
a) Bilateral corneal opacities 28. The pigment deposited in Kayser-
b) Congenital glaucoma Fleischer ring is
c) Corneal dystrophies a) Melanin
d) Keratoconus b) Haemosiderin
e) Keratoglobus c) Copper
d) None of the above
Ans. (D)
Ans. (C)
26. ‘Salmon patches’ are seen in
a) Haemorrhage into the cornea 29. Which of the following is not a
source of nutrients to cornea
b) Interstitial keratitis
a) Air
c) Retinitis pigmentosa
b) Aqueous humour
d) Phlyctenular keratitis
c) Peri-limbal capillaries
Ans. (B) as UHS exam d) Vitreous
27. A lady develops bacterial keratitis in
Ans. (D) Ref. Parsons as UHS exam
the left eye. She is started on topical
antibiotics the cornea continues to 30. A man of 30 years presents with pain
slough but ultimately the keratitis in the left eye for one week. Clinically
heals with extreme corneal thinning, there is ciliary congestion, corneal
descemetocele. This descemetocele lesion in the form of a branch and
leads to corneal perforation in 2 decreased corneal sensation. What
weeks. All of the the following is the most likely diagnosis?
except one are appropriate a) Amoebic keratitis
treatments in cases of corneal b) Dendritic keratitis
perforation:
c) Disciform keratitis
a) Amniotic membrane graft
d) Nummular keratitis
b) Bandage Contact Lens
e) Filamentary keratitis
c) Conjunctival flap
Ans. (B) as UHS exam

TOPIC #

15
SCLERA
5
1. A 55 year old lady suffers from a) Increased I0P
rheumatoid arthritis. She has b) Scleritis
developed intensely painful red left c) Injury
eye. Her Visual acuities are 6/12 both d) all of the above
eyes improving to 6/6 with pin hole.
Ans. (D) as UHS exam
In her Left eye she has red nodular
swelling approximately 2 mm from 6. Treatment of episcleritis includes
the temporal limbus. The swelling is EXCEPT
tender and fixed. It does not blanch a) Corticosteroids
with the use of phenylephrine. The b) Anti-inflammatory
most likely is: c) Analgesics
a) Scleritis d) Atropine
b) Iritis Ans. (D) as UHS exam
c) Keratitis
d) Conjunctivitis 7. The complications of scleritis
e) Choroiditis include all EXCEPT
a) Annular scleritis
Ans. (A) Ref. Renu Jogi as UHS
b) Ciliary staphyloma
exam
c) Posterior staphyloma
2. In which of the following there is d) Sclerosing keratitis
intense itching
Ans. (C) Ref. Renu Jogi as UHS
a) Mucopurulent conjunctivitis exam
b) Episcleritis
c) Scleritis 8. Intercalary staphyloma is a type of
d) Spring catarrh a) Equatorial staphyloma
b) Posterior staphyloma
Ans (D)
c) Ciliary staphyloma
3. Symptom differentiating scleritis d) Anterior staphyloma
from episcleritis is presence of
Ans (A)
a) Cornea and uveal involvement
b) Ulceration 9. The vena vorticosa exit from sclera
c) Secondary glaucoma a) At the equator
d) All of the above b) 4 mm behind the equator
c) 4 mm in front of equator
Ans. (B) Ref. Parsons as UHS exam
d) At posterior pole
4. Features of scleritis include
Ans. (B) Ref. Renu Jogi as UHS
a) Pain exam
b) Thining of sclera
c) Associated with connective tissue 10. Anterior staphyloma occurs due to
disease a) Perforating corneal ulcer
d) All of the above b) Penetrating corneal injury
c) Secondary glaucoma
Ans. (D) as UHS exam
d) All of the above
5. Common causes of staphyloma
Ans. (D) as UHS exam
include

16
CONJUNCTIVA / CORNEA UHS Solved MCQ’s Rana
Haseeb

11. The optic nerve pierces the sclera b) Conjunctiva moves freely over it
a) Anteriorly c) Hard, movable and tender
b) Posteriorly d) Cornea and uveal tract involvement
c) At the equator Ans. (D) Ref. Renu Jogi as UHS
d) 4 mm behind the equator exam
Ans. (B) Ref. Parsons as UHS exam 14. The thickness of sclera is
12. The following conditions are a) 0.5 mm
associated with blue sclerotics b) 0.1 mm
a) Deafness c) 1 mm
b) Fragilitas ossium d) 2 mm
c) Both Ans. (C)
d) None
15. Scleritis is often associated with
Ans. (C) as UHS exam a) Polyarteritis nodosa
13. The classical features of episcleritis b) SLE
include all EXCEPT c) Dermatomyositis
a) Circumscribed nodule, 2-3 mm from d) All of the above
limbus Ans. (D)

TOPIC #

17
4 LENS & CATARACT
1. Ideal site for intraocular lens c) Axial length
implantation is d) Nuclear hardness
a) In the anterior chamber e) Type of cataract
b) Transfix in the pupillary margin Ans. (C)
c) In the posterior chamber 6. Lens capsule is thinnest at the
d) Behind the posterior lens capsule a) Centre anteriorly
Ans. (C) b) Laterally
2. The crystalline lens derives its c) Superior pole
nourishment from: d) Inferior pole
a) Aqueous and vitreous humor Ans. (A)
b) Blood vessels 7. A young boy of 30 years age while
c) Cornea working in the factory suffered an
d) Connective tissue electric shock to his hands. He
survived. What could be the most
e) Zonules
possible effect on his eyes:
Ans. (A) a) Bilateral Cataracts
3. Congenital cataract is associated with b) Bilateral Dry eyes
all EXCEPT c) Bilateral Macular edema
a) Toxoplasmosis d) Bilateral Open angle glaucoma
b) Lowe’s syndrome e) Bilateral Retinal detachment
c) Galactosaemia Ans. (A) Ref. Renu Jogi as UHS exam
d) Glycogen storage disease
8. Diminished vision in daylight is seen
Ans. (D) Ref. Renu Jogi as UHS exam in
4. Which of the following is most a) Central cataract
important factor in the prevention of b) Peripheral cataract
the endophthalmitis in cataract c) Zonular cataract
surgery?
d) None of the above
a) One week antibiotic therapy prior to
surgery Ans. (A) Ref. Parsons as UHS exam
b) Preoperative preparation with 9. The test used to calculate the power of
povidone iodine intraocular lens prior to cataract
c) Trimming of eyelashes surgery:
d) Use of intravitreal antibiotics a) B /Scan
b) Biometry
Ans. (B) Ref. Parsons as UHS exam
c) FFA
5. The power of the intra-ocular lens
varies depending upon: d) HRT
a) Age of patient e) OCT
b) Amount of visual loss Ans. (B) as UHS exam

18
LENS / CATARACT UHS Solved MCQ’s Rana
Haseeb

10. The retina in case of dense cataract c) Diode laser cycloablation


can be assessed best by: d) Fistulizing operation
a) B-Scan e) Miotics and Beta blockers
b) CT Scan orbit
Ans. (A) Ref. Renu Jogi as UHS exam
c) FFA
16. Polyopia is a symptom of
d) MRI Scan
e) Retinoscope a) Cortical cataract
b) Cupuliform cataract
Ans. (A)
c) Radiation cataract
11. Cataracts are found in association
d) Electrical cataract
with
a) Parathyroid deficiency Ans. (A)
b) Myotonic dystrophy 17. The best treatment for aphakia is
c) Dinitrophenol toxicity a) Anterior chamber IOL
d) All of the above b) Contact lens
Ans. (D) as UHS exam c) Excimer laser
12. Major cause of world blindness: d) Posterior chamber IOL
a) Cataract e) Spectacles
b) Diabetic retinopathy Ans. (D) as UHS exam
c) Glaucoma
18. Following are associated with zonular
d) Onchocerciasis cataract EXCEPT
e) Trachoma a) IUGR
Ans. (A) Ref. Kanski as UHS exam b) Rickets
13. The most common complication in c) Dental anomalies
exfoliation of the lens capsule is d) Diabetes
a) Iritis Ans. (D) as UHS exam
b) Conjunctivitis
19. The etiology of complicated cataract
c) Glaucoma all, except:
d) Optic neuritis a) Disciform keratitis
Ans. (C) as UHS exam b) Iridocyclitis
14. Symptoms of cataract include all of c) Retinitis pigmentosa
the following except: d) Retinal detachment
a) Color vision defects e) Scleritis
b) Decreased vision in bright light Ans. (A) Ref. Parsons as UHS exam
c) Decreased vision in low illumination 20. The commonest cause of cataract is:
d) Glare a) Diabetes
e) Halos b) Hypo-parathyroidism
Ans. (A) Ref. Kanski as UHS exam c) Old age
15. Phakolytic glaucoma is best treated d) TORCH infections
by: e) Trauma
a) Cataract extraction Ans. (C)
b) Cyclo-destructive procedure

19
21. Following Laser is used for posterior with the movement of the eye. The
capsulotomy in posterior capsular most likely diagnosis is:
opacification: a) Asteriod Hyalosis
a) Argon laser b) Corneal Opacity
b) Carbon dioxide laser c) Nuclear Sclerosis Cataract.
c) Diode laser
d) Posterior Vitreous Detachment.
d) Excimer laser
e) Vitreous Haemorrhage
e) YAG laser
Ans. (C) Ref. Renu Jogi as UHS exam
Ans. (E)
25. Intraocular lenses are generally made
22. An otherwise fit 20 year old man
of
sustains blunt trauma to the left eye
with a large plank of wood. On a) Prolene
examination of the eye his visual b) PMMA
acuities are 6/6 and 6/6 right and left c) HEMA
eyes respectively. The visual acuity in
d) Silicone
the left eye does not improve with Pin
hole and he has a Left Relative Ans. (B) as UHS exam
Pupillary Defect. All except one can 26. A 60 year old lady presents with
cause RAPD in this patient: painless decrease in vision in the left
a) Optic Neuropathy eye. The visual acuities are 6/6 Right
b) Retinal Detachment and 6/60 in the left. The visual acuity
does not improve with pin hole. She
c) Retinal Oedema (more than 2 retinal
also has a left RAPD. All of the
quadrants)
following diseases can be a cause of
d) Sub Retinal Haemorrhage RAPD except:
(extensive)
a) Compressive Optic Neuropathy
e) Subluxated Lens
b) Nuclear Sclerotic Cataract
Ans. (E) c) End stage Glaucoma
23. After cataract operation, lenses are d) Ischaemic Optic Ncuropathy
prescribed after
c) Retinal Detachment
a) 2 weeks
Ans. (B)
b) 4 weeks
27. A 4 year old child is brought by her
c) 6 weeks
mother with bilateral white pupils.
d) 12 weeks Anterior segment and papillary
Ans. (C) as UHS exam reactions are normal. Ultrasound B-
scan was performed and was found to
24. A 70 year old gentleman who is
be normal. Most probable diagnosis
otherwise fit presents to eye OPD with
is?
progressive decrease in vision for last
2 years. His visual acuities were a) Bilateral congenital cataracts
recorded as 6/24 both eyes and b) Bilateral retinoblastoma
improved to 6/9 with pin hole. He was c) Congenital glaucoma
previously emmetropic but now his
d) Corneal dystrophies
refractive error is -2.0 diopter in each
eye. The pupillary responses, intra c) Primary hyperplastic vitreous
ocular pressures are normal. On Ans. (A) Ref. Parsons as UHS exam
distant direct ophthalmoscopy there is
a media opacity which does not move 28. Elschnig’s pearls arise from

20
LENS / CATARACT UHS Solved MCQ’s Rana
Haseeb

a) Anterior capsule of lens e) Radial keratotomy


b) Posterior capsule of lens Ans. (E)
c) Cubical cells underneath lens capsule 34. A young patient presented with rosette
d) None of the above shaped cataract which is
Ans. (C) as UHS exam characteristic of:
a) Complicated cataract
29. Phaco-Emulsification is done except
in: b) Diabetic cataract
a) Dislocated cataract c) Radiation induced cataract
b) Hyper-mature cataract d) Senile cataract
c) Immature cataract e) Traumatic cataract
d) Mature cataract Ans. (E) Ref. Renu Jogi as UHS exam
e) Morgagnian cataract
35. A patient who was operated for
Ans. (A) Ref. Renu Jogi as UHS exam cataract four months back comes to
you with foggy vision. On examination
30. Rosette-shaped cataract is a feature of
there was posterior capsular
a) Traumatic cataract thickening, which mode of treatment is
b) Diabetic cataract most useful:
c) Coronary cataract a) Argon laser
d) Complicated cataract b) Diode laser
Ans. (A) c) Excimer laser
31. The most common cause of reduced d) Krypton laser
vision in the world is: e) YAG-Laser
a) Cataract Ans. (E)
b) Diabetic retinopathy 36. The most dreadful complication of
c) Glaucoma cataract surgery on the 1st post
d) Refractive errors operative day is:
e) Trachoma a) Striate keratitis
Ans. (D) Ref. Kanski as UHS exam b) Endophthalmitis
c) Iris prolapse
32. Calculation of IOL power is called:
d) Hyphaema
a) Biometry
e) Cystoid macular edema
b) Keratometry
c) Ophthalmometry Ans. (B)
d) Pachymetry 37. Best option for treating a unilateral
cataract in a young adult with unaided
c) Tonometry
visual acuity of 6/6 in the other eye:
Ans. (A) a) Cataract surgery and correction with
33. Aphakia can be corrected by following glasses
except: b) Cataract surgery and correction with
a) Spectacles contact lens
b) Contact lenses c) Epikeratophakia
c) Intraocular lenses d) Phacoemulsification and IOL
d) Epikeratophakia implantation

21
e) Cataract surgery as such and there is no fundus reflex. What is
your diagnosis?
Ans. (D) Ref. Kanski as UHS exam
a) Orbital cellulitis
38. The only disadvantage of extra
b) Corneal ulcer
capsular cataract excision over intra
capsular cataract excision is because: c) Traumatic uveitis
a) Vitreous loss doesn't occur d) Endophthalmitis
e) Posterior uveitis
b) Retinal detachment is more common
c) Posterior capsule gets opacified Ans. (D) Ref. Parsons as UHS exam
d) We cannot implant anterior chamber 42. A 35 years female presents to the OPD
intra ocular lens (IOL) with dimness of vision to the right eye
e) Intra capsular cataract excision is the for one year. She gives history of right
more advanced technique cataract extraction but record is not
Ans. (C) as UHS exam available clinically visual acuity in
right eye is CF2m & 6/6 in left eye.
39. Burst Morgagnian cataract may cause
a) Secondary glaucoma There is now right aphakia with intact
posterior capsule. What is the most
b) Iritis
appropriate method for restoration of
c) Both
vision in right eye?
d) None
a) Spectacles
Ans. (C) Ref. Parsons as UHS exam b) Contact lenses
40. A young girl presents with history of c) Right secondary posterior chamber
difficulty in reading and focusing 10L implantation
objects at different distances. She d) Right secondary anterior chamber
feels this problem more in her right 10L implantation
eye. She also gives history of some
viral illness few weeks back. On e) Refractive surgery
examination, her visual acuity is 6/6 in Ans. (C) Ref. Renu Jogi as UHS exam
both of her eyes. Her right pupil is
43. Lensectomy is an operation whereby
larger than left. There is no direct light
reflex in right eye but very slow near a) Lens is removed
reflex is present. What is your b) Nucleus and anterior capsule are
provisional diagnosis? removed
a) Homer syndrome c) Lens and anterior vitreous phase is
b) Argyll robertson pupil removed
c) Marcus gunn pupil d) None of the above
d) Adie pupil Ans. (C) as UHS exam
e) Right physiological anisocoria
44. A 25 year old lady doctor has bilateral
Ans. (C) uveitis secondary to sarcoidosis. She
41. A small child is hit in the eye by a used was started on high dose steroid
disposable syringe by his younger therapy in 2005. Inspite of treatment
brother 02 days back. He is brought by she started to lose sight in both eyes.
his parents to the OPD with painful red In 2011 her Visual acuities were 6/36 in
eye. His eyelids are swollen. There is both eyes with no afferent pupillary
ciliary congestion, hazy cornea and defect. On distant direct
hypopyon. Pupil is yellowish white ophthalmoscopy there are bilateral
media opacities that remain stationary

22
LENS / CATARACT UHS Solved MCQ’s Rana
Haseeb

on ocular movements. The most likely cataracts. What will happen if his
diagnosis is: cataract are not removed at the
a) Age related Cataracts earliest?
b) Corneal Opacities a) He will develop lens induced
glaucoma
c) Secondary Cataracts
b) He will develop retinoblastoma
d) Retinal Detachment
c) He will develop amblyopia
e) Vitreous Opacities
d) He will develop pthysis buibi
Ans. (C) as UHS exam
e) Retinal detachment
45. In present days, the surgery for
Ans. (C) Ref. Renu Jogi as UHS exam
cataract extraction is indicated when:
49. Refractive index of lens is increased
a) Patient's age is less than 50 years
in:
b) Patient's age is more than 50 years
a) Anterior subcapsular cataract
c) When cataract reaches stage of
b) Cortical cataract
maturity
c) Morgagnion
d) When patient's vision is worse than
6/18 d) Nuclear cataract
e) Daily activities of patient are e) Posterior subcapsular cataract
disturbed Ans. (D)
Ans. (E) Ref. Kanski as UHS exam 50. The crystalline lens develops from:
46. During the 1st trimester of pregnancy, a) Surface ectoderm
the following can be a cause of b) Neuro ectoderm
congenital cataract in the baby:
c) Mesoderm
a) Abdominal and pelvic
d) Neural cest cells
ultrasonography of mother
e) Endoderm
b) Abdominal and pelvic MRI of the
mother Ans. (A) Ref. Parsons as UHS exam
c) Use of antibiotics of penicillin group 51. Diagnostic criteria of immature
by mother cataract includes
d) Rubella infection of the mother a) Greyish lens
e) Blood transfusion to the mother b) Presence of iris shadow
Ans. (D) Ref. Renu Jogi as UHS exam c) Black shadow visible against red
47. Dislocation of lens can occur because d) All of the above fundal glow
of. Ans. (C) as UHS exam
a) Retinal detachment
b) Blunt trauma
c) Whooping cough
d) Posterior scleritis
e) Vitreous haemmorhage
Ans. (B) Ref. Kanski as UHS exam
48. A one year old boy is brought to the
OPD, by his parents. Doctor
diagnosed him as having congenital

23
TOPIC #

GLAUCOMA
5
1. Treatment of choice for congenital e) Persistent hyperplastic primary
glaucoma is vitreous
a) Drugs Ans. (D)
b) Goniotomy 5. Following drugs have antiglaucoma
c) Cyclodialysis effect except:
d) Trabeculectomy a) Dipivefrin
Ans. (B) as UHS exam b) Latanorost
2. Following is the diagnostic signs of c) Pilocrpine
congenital glaucoma except; d) Timolol
a) Corneal Edema e) Tropicamide
b) Enlarge Cornea Ans. (E)
c) Haab's Striae 6. The earliest visual field defect in
d) Stocker Line POAG is:
e) Raised IOP a) Arcuate scotoma
Ans. (D) Ref. Parsons as UHS exam b) Bjerrum's scotoma
3. A man age 50-years, presented with c) Central Scotoma
glaucomatous cupping of the discs d) Paracentral scotoma
and IOP of 28 mmHg in both eyes. He e) Ring Scotoma
also gave history of using salbutamol
inhaler. Following is not an Ans. (D)
appropriate treatment option; 7. Regarding the angle of the anterior
a) Brimonidine chamber which statement is not true;
b) Dorzolamide a) The angle is bounded anteriorly by
the Schwalhe's line and posteriorly
c) Latanoprost
by ciliary body
d) Pilocarpine b) The angle is not visible to the naked
e) Timolol eye because of total internal
Ans. (E) Ref. Renu Jogi as UHS exam reflection
c) Pseudoexfoliation causes closure of
4. The differential diagnosis of
the anterior chamber angle
congenital glaucoma include all
except; d) Plateau Iris cause angle closure
a) Congenital blocked nasolacrimal e) Pigmentary glaucoma is open angle
duct Ans. (C) Ref. Kanski as UHS exam
b) Keratitis 8. Stony hard eye is seen in
c) Megalocornea a) Infantile glaucoma
d) Norrie's disease b) Chronic open angle glaucoma

24
GLAUCOMA UHS Solved MCQ’s Rana
Haseeb

c) Absolute glaucoma Ans. (D)


d) None of the above 13. The earliest visual field defect in
Ans. (C) Ref. Renu Jogi as UHS exam POAG is:
9. Regarding aqueous humor, which a) Arcuate scotoma
statement is not true: b) Bjerrums scotoma
a) ATP is required for its production c) Central Scotoma
b) Is produced by the non-pigmented d) Centro-cecal scotoma
epithelium of the ciliary body
e) Isolated paracentral nasal scotoma:
c) Levels lactate is higher than the
plasma Ans. (C)
d) Proteins content is more than 2% 14. 100 day glaucoma is seen in
e) It is IOP dependent a) Central retinal artery occlusion
Ans. (D) b) Central retinal vein occlusion
10. Treatment of choice in primary open c) Steroid induced
angle glaucoma is prime d) Primary open angle glaucoma
a) Cyclodialysis Ans. (C) Ref. Renu Jogi as UHS exam
b) Iridectomy
15. A 60 year old lady was diagnosed with
c) Cyclodiathermy primary open angle glaucoma. She
d) Medical was counseled and started on topical
anti glaucoma medication. She has a
Ans. (D) as UHS exam
history of asthma as a child. 3 days
11. A 60 years old female comes to you after starting medication she
with severe pain and visual loss in her developed shortness of breath. She
left eye for the last 2 days. On does not take any other medication. Is
examination there is corneal edema it possible that the anti glaucoma
and shallow anterior chamber with medications can precipitate an acute
raised IOP. She was diagnosed as attack of asthma and if yes then most
acute angle closure glaucoma. What is likely anti glaucoma drug to cause it
the best treatment option? is?
a) Antiglauchoma, miotics and steroids. a) Brimonidine eye drops
b) Antiglaucoma, cycloplegics and b) Dorzolamide eye drops
steroids. c) Latanoprost eye drops
c) Antiglaucoma, miotics and antibiotics d) Levobunolol eye drops
d) Antiglaucoma and antibiotics e) Pilocarpine eye drops
e) Miotics and antibiotics. Ans. (D) as UHS exam
Ans. (A) Ref. Renu Jogi as UHS exam 16. A 65 year old hypermetropic and
12. Attacks of Acute angle closure diabetic lady was examined for
glaucoma can be easily precipitated in diabetic retinopathy in the eye clinic.
a person with narrow angle by: Her visual acuities corrected with
glasses with 6/6 in both eyes. Her intra
a) Throwing bright light into his eyes
ocular pressures were recorded as
b) Putting pilocarpine drops in his eyes
15mm Hg in each eye. She underwent
c) Putting steroid drops in his eyes pupillary dilatation and was found to
d) Putting mydriatic drops in his eyes have minimal background diabetic
e) Making him sleep in a dark room retinopathy. After 2 hours she started

25
to complain of pain in both eyes and c) Primary open angle glaucoma
the Intra Ocular pressures were d) Pseudoexfolitive glaucoma
recorded as 50 mm Hg in each eye. The e) Secondary glaucoma
most likely topical drug used to dilate
the pupil and to cause this acute rise Ans. (C) Ref. Renu Jogi as UHS exam
in intra ocular pressure is?
a) Betamethasone eye drops 19. Lens induced glaucoma least possibly
will occur in:
b) Betaxolol eye drops
a) Anterior lens dislocation
c) Dexamethasone eye drops
b) Hypermature cataract
d) Pilocarpine eye drops
c) Intumescent cataract
e) Tropicamide eye drops
d) Posterior lens dislocation
Ans. (E) Ref. Parsons as UHS exam e) Posterior sub-capsular cataract
17. In congenital glaucoma with Ans. (E) Ref. Parsons as UHS exam
buphthalmos, besides intraocular
20. A 40 years old hypermetropic lady
pressure and disc, we check the
presented with severe pain and
following for diagnosis and progress
redness in her left eye since
of disease:
yesterday. On examination her visual
a) Conical diameter acuity in left eye is counting finger.
b) Visual fields The cornea is oedematous and pupil is
c) Electroretinogram mid-dilated. Her intraocular pressure
is 60mm Hg. On slit lamp examination
d) Palpebral fissure
her lens is clear. What is the most
e) B-Scan of the eye likely diagnosis?
Ans. (A) a) Acute angle closure glaucoma
18. A 60 year old gentleman comes to you b) Lens induced glaucoma
because he cannot see clearly for c) Neovascular glaucoma
distance. His present reading glasses d) Pigmentary glaucoma
are 5 years old. He is otherwise fit. His e) Primary open angle glaucoma
visual acuities 6/5 in both eyes
corrected with glasses. His refractive Ans. (A) Ref. Renu Jogi as UHS exam
error is -2 diopter spheres in both 21. Neovascular glaucoma is common in:
eyes. On examination his pupillary a) Ischaemic central retinal vein
response are normal. Anterior occlusion
segment examination is normal with
deep anterior chambers. His intra b) Vitreous hemorrhage
ocular pressure were recorded as 28 c) Hy-permature cataract
and 30 mm Hg in right and left eye d) Subluxated lens
respectively. His central corneal e) Hyphaema
thickness is 515 micron in each eye.
On direct ophthalmoscopy he has got Ans. (A)
a cupping of 0.6 in both eyes with 22. A 50 year old diabetic lady with
nerve fiber layer hemorrhage along the uncontrolled Type I diabetes and
superior margin of the left optic disc. hypertension for the last 15 years
Visual field examination shows presents with pain in the right eye.
arcuate scotoma in both eyes. The Visual acuities are counting finger in
most likely diagnosis is? the right and 6/18 in the left eye. She
a) Myopic degeneration of the retina has a right relative afferent pupillary
b) Ocular hypertension defect (RAPD) with slightly hazy

26
GLAUCOMA UHS Solved MCQ’s Rana
Haseeb

cornea on the right side. The anterior b) Congenital Glaucoma


chamber is deep with dilated blood (Buphthalmos)
vessels visible on the right iris. The c) Congenital rubella infection
eye is hard on digital tonometry. There
d) Megalocornea
is reduced red reflex on distant direct
ophthalmoscopy with vitreous e) Birth trauma to cornea
opacities in the right eye. The most Ans. (B) Ref. Parsons as UHS exam
likely cause of raised intra ocular
pressure in the right eye is? 27. A 15 years old boy with vernal
keratoconjunctivits presents with
a) Acute angle closure glaucoma
gradual loss of vision in both of his
b) Lens induced glaucoma eyes. He gives history of use of topical
c) Neovascular glaucoma dexamethasone eye drops for many
d) Pigmentary glaucoma years. On examination, his visual
e) Primary open angle glaucoma acuity in right eye was 6/12 and left eye
was 6/60. Both corneas were clear. His
Ans. (C) as UHS exam intraocular pressure was, 24mm Hg in
23. Which of the following drugs is not right eye and 30 mm Hg in left eye.
used topically for treatment of open Fundoscopy cup disc ratio was 0.5 in
angle glaucoma: right eye and 0.8 in left eye. What is the
a) Acetazolamide probable diagnosis of his visual loss?
b) Brimonidine a) Keratoconus
c) Bctoxolol b) Steroids induced Glaucoma
d) Dorzolamide c) Optic Atrophy
e) Latanoprost
d) Congenital Glaucoma
Ans. (A) Ref. Kanski as UHS exam e) Keratoglobus
24. Trabeculectomy is done for: Ans. (B) as UHS exam
a) Retinal detachment
28. Most common presenting feature of a
b) Vitreous hemorrhage patient with primary open angle
c) Glaucoma glaucoma is
d) Ptosis a) Eyeache
e) Aphakia b) Headache
Ans. (C) Ref. Renu Jogi as UHS exam c) Coloured halos
25. Peripheral anterior synechia occurs in d) Chronic deterioration of vision
noticed suddenly
a) Open angle glaucoma
b) Closed angle glaucoma Ans. (D) Ref. Kanski as UHS exam
c) Neovascular glaucoma 29. A 30 years old man presents with the
history of headache and vomiting for
d) None of the above
the last few weeks. He also gives
Ans. (B) as UHS exam history of transient loss of vision once
26. A baby age one month is brought to an or twice a day. 0/E his VA is 6/6 in both
eye clinic with findings of haziness of his eyes, pupil are reacting normal
and larger corneal size noted on both to light. His IOP is 14mm Hg in both of
sides. What do you think is the his eyes. His optic discs are swollen in
probable diagnosis in this infant? both of his eyes. What is your
provisional diagnosis in this case?
a) Congenital cataracts
a) Optic neuritis

27
b) Retinal migraine d) Give the patient immunosuppressive
c) Thrombo embolic phenomenon in drugs
retinal arteries e) Do Yag Laser Iridectomy
d) Raised intra cranial pressure
Ans. (A) Ref. Kanski as UHS exam
e) Cavernous sinus thrombosis
34. The following antiglaucama drug is
Ans. (D) Ref. Parsons as UHS exam contraindicated in a patient with
30. Regarding the risk factors of bronchial asthma:
glaucoma, which statement is not true: a) Pilocarpine
a) Age b) Acetazolamide
b) Diabetes
c) Adenaline
c) Family History
d) Beta blockers
d) Race
e) I/V Mannitol 20%
e) Smoking
Ans. (D)
Ans. (E) as UHS exam
35. A 14 years old boy from Chitral was
31. A 75 years old male has a right mature
diagnosed with Vernal
cataract for the last 2 years. He
Keratoconjunctivitis both eyes, and
refused to undergo cataract surgery.
was prescribed topical medications
One day ago he developed severe pain
which relieved his symptoms to a
in right eye and head ache. On large extent. He has come to Peshawar
examination, he has perception of for second opinion as he is losing
light in right eye, corneal edema, milky
sight in both eyes for the last one year.
white fluid in the anterior chamber and
His visual acuities are 6/60 both eyes,
IOP of 60mmHg. What is the
papillary responses are sluggish to
diagnosis? both direct and consensual light. He
a) Acute angle closure glaucoma has got giant papillae in both eyes. He
b) Lens induced glaucoma has intraocular pressures of 50 mm Hg
c) Neovascular glaucoma in each eye. On direct
d) Pigmentary glaucoma ophthalmoscopy he has got bilateral
c) Primary open angle glaucoma advanced cupping of optic disc. There
is no family history of glaucom. He
Ans. (B)
forgot to bring his medication from
32. The drug which is NOT used in open Chitral. A diagnosis of secondary
angle glaucoma glaucoma as a side effect to the topical
a) Epinephrine b) Corticosteroids drugs he was using is made. The anti
c) Beta adrenergic blocker allergic drug most likely to cause
d) Pilocarpine secondary glaucoma is:
a) Betamethasone eye drops
Ans. (B)
b) Emedastine eye drops
33. In Phacolytic glaucoma, the treatment
of choice is to control the pressure c) Lodoxamide eye drops
and then: d) Olopatadine eye drops
a) Do cataract surgery e) Sodium Cromoglycate eye drops
b) Do trabaculectomy Ans. (A) as UHS exam
c) Continue with antiglaucoma
medicine and monitor the patient

TOPIC #

28
GLAUCOMA UHS Solved MCQ’s Rana
Haseeb

NEURO-OPHTHALMOLOGY
8 (RETINA, OPTIC NERVE, OPTIC
PATHWAY)

1. Ring scotoma is seen in a) Central retinal artery occlusion


a) Papilloedema b) Central retinal vein occlusion
b) Macular oedema c) Hypertension
c) Central retinal artery occlusion d) Diabetes
d) Retinitis pigmentosa Ans. (A) as UHS exam
Ans. (D) Ref. Renu Jogi as UHS exam 6. The cardinal feature of stage 4
2. In diabetic retinopathy the new vessel hypertensive retinopathy is
formation is due to following a) Arteriovenous crossing changes
pathology: b) Flame-shaped haemorrhage
a) Retinal breaks c) Papilloedema
b) Retinal edema d) Soft exudate
c) Retinal hemorrhage
Ans. (C) Ref. Kanski as UHS exam
d) Retinal ischemia
7. A patient presented with diplopia in
e) Vitreous hemorrhage primary position along with ptosis in
Ans. (D) left eye. The eye was deviated
infrolaterally. The diagnosis will be:
3. Ophthalmoscopically, the earliest sign
of diabetic retinopathy is a) Third nerve palsy
a) Retinal haemorrhage b) Fourth nerve palsy
b) Microaneurysm c) Fifth nerve palsy
c) Soft exudate d) Sixth nerve palsy
d) Hard exudate e) Seventh nerve palsy
Ans. (B) as UHS exam Ans. (A) as UHS exam
4. The most common cause of blindness 8. A 25 year male sustains multiple
in working age population (age 16-64) fractures of the face in a car accident
is: especially superior margin of the left
orbit. After the oedema has settled the
a) Cataract
patient complains of double vision
b) Conjunctivitis during reading and walking down
c) Corneal Degeneration stairs. On examination his visual
d) Glaucoma acuities and pupils are normal. He has
left hypertropia, with head tilt to the
e) Retinal Disease right. On ocular movements testing
Ans. (E) Ref. Parsons as UHS exam the left hypertropia increases in right
5. Cattle truck appearance in fundoscopy gaze). The most likely diagnosis is:
is seen in a) Abducens Palsy

29
b) Facial Palsy undulating with the movements of the
c) Oculomotor Palsy eye and has a corrugated appearance
with extensive lattice degeneration of
d) Trochlear Palsy
the retina. The most likely diagnosis
e) Trigeminal Palsy is:
Ans. (D) Ref. Renu Jogi as UHS exam a) Vitreous Haemorrhage
9. In retinitis pigmentosa pigmentation in b) Rhegmatogenous Retinal
retina starts at Detachment
a) Posterior pole c) Serous Retinal Detachment
b) Anterior to equator d) Central Retinal artery Occlusion
c) Equator e) Central Retinal Vein Occlusion
d) The disc Ans. (B)
Ans. (C) as UHS exam 13. A 25 year male sustains multiple
10. Pathognomonic clinical sign of fractures of the face in a car accident.
proliferative diabetic retinopathy is: After the periorbital oedema has
settled down the patient complains of
a) Deep retinal hemorrhages double vision during reading and
b) Flam shaped Hemorrhages walking down stairs. On examination
c) Hard exudates his Visual acuities and Pupils are
normal. He has Left Hypertropia, with
d) Micro aneurysms
head tilt to the right. On ocular
e) Neovascularization of retina movements testing the Left
Ans. (E) Hypertropia increases in Right Gaze.
The most likely diagnosis is:
11. Nd: YAG laser wave is
a) Abducens Palsy
a) Colourless
b) Facial Palsy
b) Red
c) Oculomotor Palsy
c) Green
d) Trochlear Palsy
d) Blue
e) Trigeminal Palsy
Ans. (A) Ref. Kanski as UHS exam
Ans. (D)
12. A 60 year old hypertensive gentleman
with history of Coronary Artery 14. A 62 year old Hypertensive man with a
Bypass operation 2 years ago BMI of 31 consults an Ophthalmologist
develops sudden decrease of vision in for sudden decrease in vision in right
the left eye while performing Hajj 20 eye since this morning. His wife
days ago. His visual acuities are 6/9 complains that he snores during sleep
right eye and Hand Movements left eye and at times she feels his breathing
with no improvement with pinhole, a has stopped. His corrected visual
refractive error of -7 and -8 right and acuity is 6/36 right and 6/6 left. The
left eyes respectively and a left Visual acuity does not improve with
Relative Afferent Pupillary Defect. Pin hole and he has a Right relative
Anterior segments are normal and the afferent pupillary defect. He also has
Intraocular Pressures are within an Altitudinal Visual Field Defect in the
normal limits. On Distant Direct affected eye. The most likely diagnosis
Ophthalmoscopy there is yellow white is:
fundal reflex. On Slit lamp examination a) Anterior Ischaemic Optic Neuropathy
he has pigment cells in Vitreous. On b) Lateral Geniculate Body Infarction
Ophthalmoscopy the retina is
c) Occipital Infarct

30
NEURO-OPHTHALMOLOGY UHS Solved MCQ’s Rana
Haseeb

d) Parietal Infarct e) Isolated 6th Nerve Palsy


e) Pituitary Tumour affecting the Ans. (B) as UHS exam
Chiasma
17. A 50 year old man presents with
Ans. (A) Painless sudden decrease of vision in
15. A 30 year old lady with fair complexion the right eye. His visual acuities are
complains of sudden decrease in HM and 6/6. The visual acuity does not
vision in her right eye for the last 3 improve with pin hole. There is a Right
days. Her Visual acuities are 6/24 right Relative Afferent Pupillary Defect.
and CF close to eye and they do not Anterior segments and Intra Ocular
improve with pinhole. She has a Pressures are normal. There are no
marked relative afferent pupillary media opacities and the right fundus
defect in the right eye. She was shows a cherry red spot. All except
diagnosed as having Left one are relevant initial investigations
anisometropic amblyopia at the age of in this case:
8 years. She also complains of pain in a) Blood Pressure checking
the right eye on eye movements. On b) Carotid Auscultation
distant and direct Ophthalmoscopy c) Carotid Doppler
the is no difference between both eyes
d) Echocardiography
and they appear normal. She has a
central scotoma in the right eye on e) Vertebral Artery angiography
Visual Field examination. The most Ans. (E) Ref. Renu Jogi as UHS exam
likely diagnosis in the right eye is:
18. Candle wax spots in the retina are
a) Central Retinal artery Occlusion seen in
b) Central Retinal Vein Occlusion a) Sarcoidosis
c) Macular Oedema b) Toxocara
d) Non Arteritic Anterior Ischaemic c) Syphilis
Optic Neuropathy
d) Cytomegalo inclusion virus
e) Retrobulbar Optic Neuritis
Ans. (A) Ref. Kanski as UHS exam
Ans. (E) Ref. Parsons as UHS exam
19. A 50 Year old male from Bannu
16. A 56 year old Non-Insulin Dependent presents with rapidly decreasing
Diabetic for the last 15 years with a vision in the right eye for the last few
HbA1c of 10.4% and Blood Pressure of weeks. His vision is just perception of
190/100mmHg suddenly develops light in right eye. He was previously
Right Ptosis. The right eye is seen by multiple ophthalmologists
adducted. On ocular movement including a vitreo retinal surgeon who
examination there is failure of diagnosed him as having Inferior
adduction and elevation of Right Eye. Brach retinal Occlusion and injected
There are no pupillary abnormalities him with intra vitreal Bevacizumab
and the rest of the examination of (Avastin). On Examination he has
cranial nerves is normal with no other numerous white cells in the vitreous.
neurological deficit. The most likely There is retinitis with thick white
diagnosis is: infiltrate of retina and deep retinal
a) Pontine Infarct haemorrhages along the inferior
b) Isolated 3rd Nerve Palsy retinal vessels. The optic nerve is
swollen. He is HIV positive and has
c) Isolated 4th Nerve Palsy
CD4 count of 30. After a month of anti
d) Isolated 5th Nerve Palsy HIV therapy he develops retinal

31
detachment. The correct diagnosis both eyes. His fasting blood sugar was
was: 290 mmHg. The most common
a) Autoimmune uveitis mechanism for a diabetic patient to
lose vision is:
b) Bchcees uveitis
a) Cataract
c) Cytomegalovirus Retinitis
b) Macular oedema
d) Retinal Toxoplasmosis
c) Vitreus hemorrhage
e) Retinitis Proliferans d) Retinal detachment
Ans. (C) e) Opaque membranes in vitreous
20. The commonest presenting sign in Ans. (B)
retinoblastoma is:
24. Retina after death becomes
a) White papillary reflex
a) Transparent
b) Dilated pupil
b) White
c) Proptosis
c) Black
d) Squint
d) Red
e) Uveitis
Ans. (B) as UHS exam
Ans. (A) as UHS exam
25. A 21/2 years old male child is brought
21. A 66 year old Non Insulin dependent by his parents with "Cat Eye Reflex" in
diabetic for the last 15 years with a his left eye for last 1 month. Nov they
HBA1c of 10.4% and blood pressure of feel that affected eye is bigger than the
190/100mmHg suddenly develops other eye. On examination there is
right ptosis. The right eye is abducted. mild congestion and pupillary reflex is
On ocular movement examination sluggish. One of his siblings has
there is failure of adduction and undergone left enucleation for almost
elevation of right eye. There are no same type of eye disease. What is the
pupillary abnormalities and the rest of most probable diagnosis?
the examination of cranial nerves is
a) Left congenital cataract
normal with no other neurological
defcit. The most likely diagnosis is: b) Left congenital glaucoma
a) Pontine Infarct c) Left Persistent Primary Hyperplastic
Vitreous (PHPV)
b) Isolated 3rd Nerve Palsy
c) Isolated 4th Nerve Palsy d) Left retinoblastoma
d) Isolated 5th Nerve Palsy e) Left traumatic cataract
e) Isolated 6th Nerve Palsy Ans. (D) Ref. Kanski as UHS exam
Ans. (B) Ref. Kanski as UHS exam 26. A 36 years female presents with
decrease in visual acuity in the left eye
22. Flame-shaped haemorrhages are seen for one week. Visual acuity is 6/6 in
commonly in the retinopathy of right eye and 6/60 in left. Colour vision
a) Diabetes is markedly reduced with RAPD in
b) Hypertension effected eye. Fundi are within normal
c) Retinitis pigmentosa limits. The most reliable investigation
d) All of the above for diagnosis of this conditions is:
a) Perimetry
Ans. (B) as UHS exam
b) Visual evoked potential
23. A 44 years male presents with
dimness of vision for the last six c) Blood complete and ESR
months. His visual acuity was 6/18 in d) Serological tests

32
NEURO-OPHTHALMOLOGY UHS Solved MCQ’s Rana
Haseeb

e) MRI d) Subluxated lens.


Ans. (B) Ref. Renu Jogi as UHS exam e) Hyphaema
27. An ophthalmologist is demonstrating Ans. (A) Ref. Renu Jogi as UHS exam
pupillary light reflexes with the house 31. The mother of a one and a half year old
officers. Regarding relative afferent child gives history of a white reflex
papillary defect, it is the result of a from one eye for the past 1 month. On
lesion in: computed tomography scan of the
a) 3rd Nerve orbit there is calcification seen within
b) Optic Nerve the globe. Most likely
diagnosis is:
c) Pretectal nucleous
a) Coats disease
d) Ciliary ganglion b) Congenital cataract
e) Sphincter pupillae muscle c) Endophtalmitis
Ans. (B) d) Fabry's disease
e) Retinoblastoma
28. Rhegmatogenous retinal detachment
is due to Ans. (E)
a) Tumour 32. A 30-year-old female presents to the
b) Retinal break eye clinic with an acute history of mild
c) Vitreous traction pain on eye movements and blurring
in the right eye. Examination reveals a
d) Proliferative retinopathy visual acuity of 6/36 in the right eye but
Ans. (B) Ref. Kanski as UHS exam 6/6 in the left eye, a central scotoma in
the right eye, with a right swollen optic
29. A 55 years old diabetic lady presents
disc. What is the most likely
with the complaint of horizontal
diagnosis?
diplopia for the last few days. Her
diplopia gets worse while looking a) Central Retinal Artery Occlusion
towards the right. On examination she b) Central Retinal Vein Occlusion
is having 6/9 corrected VA in her both c) Optic Neuritis
eyes. She is also having right
d) Papilloedema
convergent squint. Her discs are
normal and she is having back ground e) Retinal detachment
diabetic retinopathy. What is the most Ans. (C) Ref. Kanski as UHS exam
probable cause for her diplopia?
33. When should a case of Non-Insulin
a) Right 3rd nerve palsy dependent diabetes mellitus (NIDDM)
b) Diabetic retinopathy with a history of diabetes for one year
c) Right 6th nerve palsy have an ophthalmic examination?
d) External ophthalmoplegia a) As early as feasible
e) Right temporal retinal detachment b) After one year
Ans. (C) as UHS exam c) After 5 years
d) After 10 years
30. Neovascular glaucoma is common in:
e) Only after visual symptoms develop
a) Ischaemic central retinal vein
occlusion Ans. (A) Ref. Parsons as UHS exam
b) Vitreous haemmorhage 34. Oculomotor nerve palsy causes all
c) Hypermature cataract except:

33
a) Diplopia vision in the right eye associated with
b) Downward deviation severe pain over the Right Temple. His
Visual Acuities are Perception of Light
c) Miosis
and 6/9 Right and Left Eyes
d) Outward eye deviation respectively. The Visual Acuity does
e) Ptosis not improve with Pin hole in the Right
Eye but improves to 6/6 in the left. He
Ans. (C)
has Right Relative Afferent Pupillary
35. Relative Afferent Papillary Defect Defect. No abnormality is detectable
(RAPD) is seen in: on Anterior segment Examination and
a) A mature cataract Distant Direct Ophthalmoscopy. On
Direct Ophthalmoscopy the Right
b) Hypertensive retinopathy
retina is pale, swollen and has
c) Retinal detachment attenuated retinal vessels devoid of
d) Viral Keratitis blood. There is also a cherry red spot
c) Vitamin A deficiency on the macula. On Examination of the
Scalp the Frontal branch of Superficial
Ans. (C) Ref. Kanski as UHS exam Temporal artery is excruciatingly
36. Bi-temporal hemianopia is a tender. His ESR was noted to be 80mm
characteristic feature of? in the first hour. The most likely
a) Glaucoma diagnosis is:
b) Optic neuritis a) Non Arteritic Anterior Ischaernic
c) Papilloedema Optic Neuropathy
d) Pituitary tumor b) Central Retinal Artery Occlusion
e) Retinitis pigmentosa c) Central Retinal Vein Occlusion
Ans. (D) d) Macular Degeneration
37. According to the latest population c) Vitreous Haemorrhage secondary to
based survey conducted in Pakistan, Posterior Vitreous Detachment
what is the blindness rate in Pakistan Ans. (B) as UHS exam
a) 0.50%
40. A 30 years old male with high myophia
b) 0.90% comes to the eye OPD with a 2 weeks
c) 1.5% history of photopsia, decrease vision
d) 1.78% an floaters in his left eye. On
examination his VA is CF and there is
e) 2.0%
relative afferent pupilary defect and
Ans. (B) tobacco dust in the vitrous. Which one
38. The most common cause of visual of the following is the most accurate
impairment in diabetic retinopathy is diagnosis?
a) Advance diabetic eye disease a) Chorodial detachment
b) Background Diabetic Retinopathy b) Exudative R/D
c) Diabetic Macular edema c) Rhegmatogenous R/D
d) Severe non-proliferative Diabetic d) Retinoschisis.
Retinopathy e) Tractional R/D
e) Proliferative Diabetic Retinopathy Ans. (C) Ref. Parsons as UHS exam
Ans. (C) Ref. Renu Jogi as UHS exam 41. A 7 years old child is brought by his
39. A 70 year old Hypertensive gentleman mother with leuco-coria in his right
develops sudden profound loss of eye for the last one month. According

34
NEURO-OPHTHALMOLOGY UHS Solved MCQ’s Rana
Haseeb

to the mother there is a history of long b) High myopia


contacts with puppies. What is your c) Diabetic retinopathy
most probable diagnosis?
d) All of the above
a) Congenital cataract
Ans. (D) as UHS exam
b) Persrstent primary hypaplastic
vitrous. 46. A fit 48 years old woman complains of
c) Retino blastoma a very severe headache and droopy
left upper lid she gets double vision
d) Retinopathy of prematurity when she lifts up her eye lid:
e) Toxocaric granuloma a) Fifth cranial nerve palsy
Ans. (E) b) Fourth cranial nerve palsy
42. A 70 years male comes to the eye c) Seventh cranial nerve palsy
department with a visual loss in his d) Sixth cranial nerve palsy
right eye. On examination his vision
e) Third cranial nerve palsy
was reduced to no light perception
with afferent papillary defect. Fundus Ans. (E) Ref. Renu Jogi as UHS exam
shows diffuse retinal edema and 47. A 35 year old gentleman with Type 1
cherry red spot at macula. What is the Diabetes for the last 20 years notices a
most probable diagnosis? Sudden Painless Decreases in vision
a) Anterior ischemic optic neuropathy. in the Left Eye for the 2 days. His
b) Central retinal vein occlusion. visual acuities are 6/12 Right and 6/60
c) Central retinal artery occlusion Left eye. The visual acuity improves to
6/6 in the Right but does not improve
d) Nutritional optic neuropathy in the left eye with pinhole. He has a
e) Optic neuritis. left Relative Afferent Pupillary Defect.
Ans. (C) as UHS exam On distant direct Opacity there is no
Media opacity. On direct
43. Left homonymous hemianopia may be Ophthalmoscopy there is an elevated
caused by which of the following "boat shaped" stationary
lesion of : haemorrhage approximately 4 disc
a) Left optic nerve diameters wide with sharply
b) Optic chiasma demarcated boundaries obscuring the
macula and retinal blood vessels. The
c) Left optic tract diagnosis is:
d) Right Parietal lobe
a) Vitreous Haemorrhage
e) Calcanine fissure
b) Sub Retinal Haemorrhage
Ans. (C) c) Sub Macular Haemorrhage
44. Salt and pepper appearance of fundus d) Pre Retinal Haemorrhage
is seen in e) Deep Retinal Haemorrhage
a) Leprosy
Ans. (D) as UHS exam
b) Retinitis pigmentosa
48. An 80 years old Caucasian female
c) Congenital syphilis complains of recent problems with
d) Toxoplasmosis reading vision, specifically words
Ans. (C) appearing distorted and blank patches
being present. The most probable
45. Retinal detachment is associated with cause is:
a) Malignant melanoma a) Acute angle closure Glaucoma

35
b) Age-related macular degeneration a) Second
c) Anterior ischemic optic neuropathy b) Third
d) Cataract c) Fourth
e) Central retinal artery occlusion d) Sixth
Ans. (B) e) Seventh
49. Pigmentary retinal dystrophy is Ans. (E)
associated with 53. Cherry red spot in the macular area of
a) Laurence-Moon-Biedl syndrome the retina is seen in:
b) Sturge-Weber syndrome a) Age related macular degeneration
c) Reiter’s disease b) Central chorio retinitis
d) Von Recklinghausen disease c) Central retinal artery occlusion
Ans. (A) Ref. Kanski as UHS exam d) Central retinal vein occlusion
50. A 25 years female presented to the eye e) Central serous retinopathy
OPD with history of sudden loss of Ans. (C)
vision in her right eye. On examination
54. 30 year old lady with fair complexion
her VA is 6/60 In the effected eye and
complains of sudden decrease in
there is relative afferent papillary
vision in her Right eye for the last 3
defect and defective color vision with
days with dull ache in the affected eye.
normal fundus. What is your
Her Visual acuities are 6/24 Right and
diagnosis?
CF close to eye. The visual acuity does
a) Amblyopia not improve with pinhole. She has a
b) Anterior ischemic optic neuropathy marked Relative afferent pupillary
c) Leber hereditary optic neuropathy defect in the Right eye. She was
diagnosed as having Left
d) Retrobulbar optic neuritis.
anisometropic amblyopia at the age of
e) Toxic optic neuropathy 8 years. She also complains of pain in
Ans. (D) the Right Eye on eye movements. On
distant and direct Ophthalmoscopy
51. A 70 year old diabetic and the Optic Discs; Retinae and its blood
hypertensive man complains of not vessels appear normal. She has a
being able to see on his right side. On central scotoma in the Right Eye on
examination his Visual acuities are 6/6 Visual Field examination. The most
both eyes with no Relative Afferent likely diagnosis is:
Pupillary Defect. He has Right
Hemianopia on Visual field a) Right Central Retinal artery
examination. In this patient the most Occlusion
unlikely diagnosis is: b) Right Central Retinal Vein Occlusion
a) Optic Neuritis Right Eye c) Right Macular Oedema
b) Ischaemia of Optic Radiation d) Right Non Arteritic Anterior
c) Ischaemia of Lateral Geniculate Ischaemic Optic Neuropathy
Body e) Right Retrobulbar Optic Neuritis
d) Occipital Lobe Infarct Ans. (E) Ref. Parsons as UHS exam
e) Parietal Lobe Infarct 55. A tuberculous patient on treatment
Ans. (A) Ref. Parsons as UHS exam complaining of decreased vision. O/E
he has mild optic disc swelling and a
52. Tarsorrhaphy is indicated in which of visual acuity of 6/36 in both eyes and
the following nerve palsy:

36
NEURO-OPHTHALMOLOGY UHS Solved MCQ’s Rana
Haseeb

Red/Green color deficiency. Which Ans. (A) Ref. Renu Jogi as UHS exam
drugs is more likely to cause this:
58. Central Retinal Artery occlusion is
a) Ethambutol characterized by:
b) Isoniazid a) Afferent pupillary defect
c) Pyrizinamide b) Disc edema
d) Rifampicin c) Flame shaped hemorrhages
e) Streptomycin d) Venous tortuosity
Ans. (A) e) Vitreous hemorrhage
56. A man 65-year of age was operated on Ans. (A) as UHS exam
his right eye for cataract with a vision
of CF. Next day his visual acuity 59. The most common primary orbital
remain the same. Following may be malignancy in childhood is:
the reason for his poor visual acuity a) Capillary Hemangioma
except; b) Leukemia
a) Age related macular degeneration c) Lymphangioma
b) Corneal Edema d) Rhabdomyosarcoma
c) Dilated pupil e) Retinaoblastoma
d) Incorrect Biometry Ans. (E) Ref. Renu Jogi as UHS exam
e) Retinal detachment
60. In Bell's palsy, the cornea is affected
Ans. (A) Ref. Kanski as UHS exam because of:
57. A 62 year old Hypertensive man with a a) Entropion
BMI of 31 consults an Ophthalmologist b) Trichiasis
for Sudden Painless Decrease in c) Loss of corneal sensations
vision in right eye since this morning.
His wife complains that he snores d) Exposure keratitis
during sleep and at times she feels his e) Severe watering from the eye
breathing has stopped. His corrected Ans. (D) Ref. Kanski as UHS exam
Visual acuity is 6/36 right and 6/6 left.
The Visual acuity does not improve 61. Toxoplasmosis usually affects
with Pin hole and he has a right a) Iris
Relative Afferent Pupillary Defect. On b) Ciliary body
direct Ophthalmoscopy the Inferior
c) Macula
half of right optic disc is swollen. He
also has a Superior Altitudinal Visual d) Ora serrata
Field Defect in the affected eye. His Ans. (C) Ref. Kanski as UHS exam
Fasting Lipid Profile shows
Hyperlipidaemia. The most likely 62. A 40 year old lady with a BMI of 30 and
diagnosis is: using oral contraceptives presents
with transient obscurations of vision
a) Anterior Ischaemic Optic Neuropathy
for last 1 month. She has also
b) Central Retinal Artery Occlusion complains of headache for the last
c) Papilloedema month. On examination her visual
d) Pituitary Tumour affecting the acuities are 6/6 both eyes with no
Chiasma Pupillary deficit. The Intra ocular
pressures and distant direct
e) Optic Neuritis Ophthalmoscopy are normal. On

37
direct Ophthalmoscopy both of her following investigations is appropriate
optic nerves are swollen with absent in this case:
optic cup. The Nerve fibre layer has a) Automated Visual Field
multiple haemorrhages and cotton Examination.
wool spots. The most likely diagnosis
b) Plain X Ray of Pituitary
is:
c) MRI of Pituitary.
a) Bilateral Ant Ischaemic Optic
Neuropathy d) CT of Pituitary
b) Bilateral Central Retinal Vein e) Ocular Coherence Tomography of
Occlusion. Macula.
c) Bilateral Retinal Artery Occlusion. Ans. (E) Ref. Parsons as UHS exam
d) Bilateral Retinal Vasculiti 66. A 40 year old lady with papilloedema
e) Papilloedema underwent lumbar puncture to check
her CSF pressure. After the procedure
Ans. (B) she complains of postural headache
63. In retinoblastoma, the commonest and develops failure of abduction of
complaint of parents at the time of left eye. The likely diagnosis is:
presentation about their child is: a) Brainstem Infarction
a) Buphthalmos b) Cerebral Infarction
b) Microphthalmia c) Coning of the Brain stem with Left 6th
c) Exophthalmos Nerve Palsy
d) Leukocoria d) Medial Longitudinal Fascicle
e) Sticky discharge from the eyes Infarction
e) Ventricular Haemorrhage
Ans. (D) as UHS exam
64. The most common cause of papillitis Ans. (C)
is 67. A 30 year gentleman complains of
a) Herpes zoster sudden painless decrease of vision in
the Right Eye for the last 4 days. The
b) Multiple sclerosis
Unaided vision is CF Right and 6/36
c) Uveitis Left. The vision improves to 6/6 with -
d) Diabetes mellitus 5 Diopter Correction in the Left eye
however if does not improve in the
Ans. (B) as UHS exam
right eye either with refraction or pin
65. A 30 year old hospital worker presents hole. He has a Right Afferent Pupillary
to the eye outpatients department with Defect. On slit lamp Examination there
decreased vision in the left eye for last are a few white blood cells in the
3 days. He is examined by an Vitreous. On Direct Ophthalmoscopy
ophthalmologist. The visual acuities he has large retinal detachment
are 6/6 and 6/12 right and left eyes centered on the macula extending 3
respectively. He is told that he has disc diameters on each side. In
"normal" sight. He presents the addition there are a few other discrete
following morning to another retinal detachments in the right eye.
ophthalmologist who notices that he He recovered from Flu a week ago. The
has Left Relative Afferent Pupillary most likely diagnosis is:-
Defect and on Confrontation visual a) Valsalva Retinopathy
field examination he has Bitemporal
Heminaopia. The left eye shows mild b) Rhegmatogenous Retinal
optic disc pallor with clinically normal Detachment
maculae. All except one of the c) Serous Retinal Detachment

38
NEURO-OPHTHALMOLOGY UHS Solved MCQ’s Rana
Haseeb

d) Central Retinal Artery Occlusion He has been wearing glasses since the
e) Central Retinal Vein Occlusion age of 6 years and his refractive errors
were recorded as -6 and -8 Dioptres
Ans. (C) Right and left eyes respectively a year
68. Enlargement of blind spot is a sign of ago. Visual Field examination shows
a) Avulsion of optic nerve an Inferior altitudinal defect. The Intra
ocular pressures are 10 and 5mm Hg
b) Papillitis
right and left eyes respectively. There
c) Papilloedema is bilateral vitreous degeneration with
d) Retinal detachment pigment cells on microscopy in the left
Ans. (C) as UHS exam eye. Direct Ophthalmoscopy reveals
undulating retina. The most likely
69. Consecutive optic atrophy occurs diagnosis is:
following
a) Left Anterior Ischaemic Optic
a) Retinitis pigmentosa Neuropathy
b) Central retinal artery occlusion b) Left Central retinal artery Occlusion.
c) Both c) Left Central Retinal Vein Occlusion.
d) None d) Left Rhegmatogenous Retinal
Ans. (C) Detachment.
70. The optic nerve extends upto e) Left Posterior Vitreous Detachment
a) Optic chiasma with Vitreous Haemorrhage.
b) Optic tract Ans. (D) Ref. Renu Jogi as UHS exam
c) Lateral geniculate body 73. At the pre-proliferative stage of
d) Optic radiation diabetic retinopathy, the best option
Ans. (A) as UHS exam for preventing the progress of disease
is:
71. In retinitis pigmentosa, the
a) YAG laser capsulotomy
pathognomonic sign on fundus
examination is: b) Good metabolic control of diabetes
a) Cotton wool spots c) Pan retinal photocoagulation
b) Hard exudates d) Subconjuctival injection of steroids
c) Micro aneurysms e) Vitrectomy and injection of silicone
oil
d) Bone spicule pigmentation along the
blood vessels Ans. (B)
e) Neovascularization around the disc 74. Papilloedema can be differentiated
and in periphery from papillitis by the following
Ans. (D) Ref. Parsons as UHS exam features
a. gradual onset with slow progress b.
72. A 30 year old bank clerk who is
bilateral c. other signs of central nervous
otherwise healthy presents with
system involvement d. all of the above
Sudden Painless decrease of vision in
the Left eye since this morning. The Ans. (D) as UHS exam
vision in the left eye is 6/60 and does 75. Which of the following is the main
not improve with pinhole. He has a Left cause of neovascular glaucoma?
Relative Afferent Pupillary Defect. For
the last 1 week he has noticed a) Central retinal artery occlusion
Photopsia and Floaters in the Left eye. b) Branch retinal artery occlusion

39
c) Central retinal vein occlusion a) Retrobulbar neuritis
e) Hypertension b) Papillitis
d) Branch retinal vein occlusion c) Toxic amblyopias
Ans. (C) d) Papilloedema
76. Differential diagnosis of retrobulbar Ans. (C) as UHS exam
neuritis includes 81. A 30 year gentleman complains of
a) Hysteria sudden loss of vision in the right eye
b) Malingering for the last 4 days. The Unaided vision
is CF right and 6/36 left. The vision in
c) Both
the left eye improves to 6/6 with -5
d) None Diopter Correction. The visual acuity
Ans. (C) Ref. Kanski as UHS exam does not improve in the right eye
either with refraction or pin hole. He
77. A high myope young adult presents has a marked relative afferent
with rapid loss in the nasal field of his pupillary defect. On slit lamp
vision. It was preceeded by flashes examination there are white blood
and floaters. He also gives history of cells in the vitreous. On direct
trivial trauma in playground, few days ophthalmoscopy he has large retinal
back. What can be the cause of his detachment elevating the macula and
problem? extending 3 disc diameters on each
a) Traumatic cataract side. In addition there are a few other
b) Vitreous hemorrhage discrete retinal detachments in the
right eye. The most likely diagnosis is:
c) Keratoconus
a) Vitreous Haemorrhage
d) Rhegmatogenous retinal
detachment b) Rhegmatogenous Retinal
Detachment
e) Change in his refractive error
c) Serous Retinal Detachment
Ans. (D) Ref. Renu Jogi as UHS exam
d) Central Retinal artery Occlusion
78. Synoptophore is used for all the
e) Central Retinal Vein Occlusion
following except:
a) Color blindness Ans. (C) Ref. Renu Jogi as UHS exam
b) Depth perception 82. Yellowish waxy disc is seen typically
in
c) Abnormal retinal correspondence
a) Retinal detachment
d) Esotropia
b) Retinitis pigmentosa
e) Exotropia
c) Primary optic atrophy
Ans. (A)
d) Postneuritic optic atrophy
79. The treatment of retrobulbar neuritis
includes all EXCEPT Ans. (B)
a) Retrobulbar injection of 83. A 35 year old gentleman with Type 1
dexamethasone Diabetes for the last 20 years notices a
sudden decreases in vision in the left
b) Antibiotics
eye for the 2 days. His visual acuities
c) Vitamin B1, B6 and B12 are 6/12 Right and 6/60 Left eye. The
d) Vasodilators visual acuity improves to 6/6 in the
Ans. (B) right but does not improve in the left
eye with pinhole. He has a left Relative
80. Marcus Gunn pupil is diagnostic of Afferent Pupillary Defect. On distant

40
NEURO-OPHTHALMOLOGY UHS Solved MCQ’s Rana
Haseeb

direct Opacity there is no Media Ans. (A)


opacity. On direct Ophthalmoscopy
there is a "boat shaped" haemorrhage 86. Secondary optic atrophy occurs
with sharp demarcation obscuring the a) Following injury or direct pressure to
macula and blood vessels. The the optic nerve
diagnosis is: b) Following extensive retinal disease
a) Vitreous Haemorrhage c) Following papilloedema and papillitis
b) Sub Retinal Haemorrhage d) Following central retinal artery
c) Sub Macular Haemorrhage occlusion
d) Sub Hyaloid Haemorrhage Ans. (A) as UHS exam
e) Deep Retinal Haemorrhage
Ans. (D) Ref. Parsons as UHS exam
84. The common causes of papilloedema
include
a) Intracranial tumour
b) Grade 4 hypertensive retinopathy
c) Subdural haematoma
d) All of the above
Ans. (D) as UHS exam
85. A 20 year old male who is Type 1
diabetic for the last 15 years develops
severe dry cough. During an
particularly severe episode of
coughing he suddenly develops
painless sudden decrease in vision in
both eyes. The visual acuities are 6/60
both eyes with no improvement with
pinhole. On examination there is no
relative afferent pupillary defect and
intra ocular pressure were recorded as
23mm Hg in both eyes. The Red reflex
is markedly reduced in both eyes on
distant direct ophthalmoscopy and
there are media opacities which move
against the movement of eye. On
Direct Ophthalmoscopy retinal
neovascularisation is noted in the
periphery of both retinae but the
details of the central retina are
obscured. The most likely cause of
reduced red reflex is:
a) Vitreous Haemorrhage
b) Retinal Vein Occlusion
c) Retinal Artery Occlusion
d) Optic Neuropathy
e) Cataract

41
TOPIC #

ORBIT / ANATOMY /
9 OCULAR TRAUMA
1. A 13 years old boy is brought to an assymetrical axial proptosis with lid
ophthalmologist with pain and diffuse retraction and lid lag. CT-Scan shows
swelling of right upper lid. On enlargement of recti muscles. Which is
examination the boy is febrile and the the most probable diagnosis?
upper lid area is red, swollen and a) Cavernous haemengioma
tender. What is the most likely
b) Orbital pseudotumour
diagnosis?
c) Orbital cellulitits.
a) Amyloidosis of the lids
d) Optic nerve meningioma
b) Preseptal Cellulitis
e) Thyroid eye disease.
c) Posterior Blepharitis
d) Seborrheic Blepharitis Ans. (E) Ref. Parsons as UHS exam
e) Ulcerative Blepharitis 5. Orbital Cellulitis:
Ans. (B) a) Does not cause motility disturbance
b) Frequently causes intracranial
2. Axial Proptosis occurs in:
infection in children
a) Frontal mucocele
c) Gentamycin is an appropriate
b) Lacrimal gland tumors antibiotic
c) Maxillary carcinoma d) Is most frequently caused sinus
d) Orbital floor fracture infection
e) Thyroid ophthalmopathy e) Is usually not accompanied by fever
Ans. (E) as UHS exam Ans. (D)
3. Regarding the risk factors of ocular 6. The management of a case of a
trauma, which statement is not suspected intra-ocular foreign body
correct: includes all except;
a) More common in females a) A-Scan of the eye
b) More common in low education b) B-Scan of the Eye
group c) CT-Scan of the Orbit
c) More common in low income group d) MRI-Orbits
d) More common in third world c) X-Ray skull
countries
Ans. (D) Ref. Renu Jogi as UHS exam
e) More common in old age
7. Pulsating proptosis is seen in
Ans. (E)
a) Orbital varicose vein
4. A 40 years old female comes to the eye
b) Arteriovenous aneurysm
OPD with prominent eyes, grittiness
and foreign body sensions in her both c) Cavernous sinus thrombosis
eyes for the last 6 months. On d) Thyrotoxicosis
examination there is bilateral
Ans. (B) Ref. Renu Jogi as UHS exam

42
OCULAR TRAUMA UHS Solved MCQ’s Rana
Haseeb

8. Ultrasonography is helpful in and uncomplicated repeated lumbar


confirming the diagnosis of: punctures reveals numerous RBCs in
a) Central retinal artery occlusion CSF. The likely diagnosis is an
expanding and leaking aneurysm in:
b) Central retinal vein occlusion
a) Frontal lobe
c) Retinitis pigmentosa
b) Interpenduncular fossa
d) Subluxated clear crystalline lens
c) Occipital lobe
e) Thyroid Ophthalmopathy
d) Parietal lobe
Ans. (E)
e) Temporal lobe
9. A patient presented with unilateral
Ans. (B)
proptosis which was compressible
and increases on bending forward. No 12. A 45 year old diabetic patient with a
thrill or bruit was present. MRI shows HbAlc of 14.5% develops a boil on left
a retro-orbital mass with enhance- side of nose. 2 days later he presents
ment. The likely diagnosis is:- with pain, chemosis and decreased
a) AV malformations vision in the left eye, he also had
severe headache, nausea and
b) Fracture roof the orbit
vomiting. On examination his Visual
c) Neurofibromatosis acuities are 6/6 right and counting
d) Orbital varix fingers Left. The left eye movements
e) Orbital encephalococle are restricted in all gazes and there is
numbness in the upper divisions of
Ans. (D) the 5th nerve. His left pupil is mid
10. Lid retraction is a characteristic sign dilated and also has a left relative
of: afferent pupillary defect. On
fundoscopy the left optic nerve is
a) Acute facial nerve palsy
swollen. His body temperature is 101F.
b) Myasthenia gravis After 24 hours the right eye develops
c) Myotonic dystrophy similar features. The likely diagnosis
d) Third nerve palsy is:
e) Thyrotoxicosis a) Pharyngo conjucntival fever
Ans. (E) as UHS exam b) Cavernous sinus Thrombosis
c) Orbital Cellulitis
11. A 70 year old hypertensive lady is
brought by her relatives to casualty d) Preseptal Cellulitis
medical officer with increasing e) Tolosa Hunt Syndrome
confusion and severe headache. Her Ans. (B) Ref. Parsons as UHS exam
pulse is 90/min and her BP is
240/120mmHg. She knows her name 13. A child has had recurrent episodes of
but does not recognise that she is in right ear ache and discharge along
the hospital. On examination she has with fever for the last 1 year. For last 2
slight Neck rigidity but her days he has developed ipsilateral
temperature is 98.6 F. Her right pupil is periorbital pain, ipsilateral failure of
semi dilated but ocular movements are abduction of right eye and ipsilateral
normal. She is admitted for control of weakness of facial muscles. The most
her blood pressure. After 1 hour she likely site of infection is:
develops right ptosis and on lifting the a) Cavernous sinus infection
eye lid the right eye is abducted. After b) Frontal Sinusitis.
an another 1 hour she drifts into coma

43
c) Orbital Cellulitis. check. His corrected visual acuities
d) Petrous part of Temporal bone are 6/6 both eyes with refractive error
of -4.0 dioptres both eyes. His intra
e) Sphenoidal sinusitis.
ocular pressures were recorded as
Ans. (D) 27mmHg both eyes. There is no
14. A 14 year old child is reviewed 7 days relative Afferent Pupillary Defect. The
after developing Traumatic Hyphaema optic discs arc normal with no visual
secondary to blunt Trauma while field defect. The most likely diagnosis
playing Squash. His Visual acuities, is:
Pupillary reflexes are normal. Slit a) Myopic degeneration of the retina
Lamp Examination did not reveal any b) Ocular hypertension
abnormality. It is mandatory to carry c) Primary open angle glaucoma d.
out the following except:
d) Pscudoexfolitive glaucoma
a) Discharge without any further tests.
e) Secondary glaucoma
b) Counseling regarding the use of
safety goggles. Ans. (B)
c) Intra ocular pressure check 18. A 30 year old man accidentally
d) Indirect ophthalmoscopy along with splashes his eyes with a freshly made
scleral indentation solution of quick lime. He is intensely
photophobic and cannot open his
e) Gonioscopy
eyes properly. He presents to you in a
Ans. (A) Ref. Kanski as UHS exam BHU where you are working as
15. The first line of treatment in chemical Medical Officer. What first aid
injury is: treatment would you carry out for this
gentleman?
a) Oral analgesia
a) Intravitreal antibiotics
b) Saline irrigation
c) Topical antibiotics b) Oral antibiotics
d) Topical cycloplegia c) Topical broad spectrum antibiotic
e) Topical steroid d) Topical steroids
Ans. (B) Ref. Renu Jogi as UHS exam e) Wash the eye copiously with Normal
saline
16. A 55 year old man is found to have
Normal visual acuity and normal intra Ans. (E)
ocular pressures but he has bilateral 19. In blow out fracture the commonest
optic disc cupping and automated bone to fracture is:
visual field analysis shows bilateral a) Ethmoidal (medial wall)
arcuate scotomas. The anterior
segments and retinae are otherwise b) Frontal (roof)
normal. The most likely diagnosis is: c) Maxillary (floor)
a) Anterior ischaemic optic neuropathy d) Lachrymal (medial wall)
b) Ocular hypertension e) Zygomatic (lateral wall)
c) Normal tension glaucoma Ans. (C) as UHS exam
d) Macular oedema 20. The most common cause of proptosis
e) Retinal detachment is:
Ans. (C) Ref. Parsons as UHS exam a) Orbital hemorrhage
b) Orbital infection
17. A 70 year old gentleman visits an
c) Orbital tumor
optometrist for a routine refraction
d) Orbital pseudo-tumor

44
OCULAR TRAUMA UHS Solved MCQ’s Rana
Haseeb

c) Thyroid ophthalmopathy Vision in the left eye, he also had


severe headache, nausea and
Ans. (E) Ref. Kanski as UHS exam
vomiting. On examination his visual
21. A patient have blunt trauma with acuities are 6/6 Right and Counting
tennis ball and having hyphema, fingers Left. The left eye movements
which is: are restricted in all gazes and there is
a) Blood in anterior chamber numbness in the upper divisions of
b) Cells in anterior chamber the 5th nerve. His left pupil is mid
dilated and also has a Left Relative
c) Foreign body in anterior chamber Afferent Pupillary defect. On
d) Protein in anterior chamber Fundoscopy the left optic nerve is
e) Pus in anterior chamber swollen. His body temperature is 101F.
After 24 hours the right eye develops
Ans. (A) similar features. The likely diagnosis
22. A child has had recurrent episodes of is:
right ear ache and discharge along a) Pharyngo conjucntival fever
with fever for the last 1 year. For last 2
b) Cavernous sinus thrombosis
days he has developed ipsilateral
periobital pain, ipsilateral failure of c) Orbital cellulitis.
abduction of right eye and ipsilateral d) Preseptal cellulitis
weakness of facial muscles. The most e) Tolosa hunt syndrome
likely site of infection is:
Ans. (B) Ref. Renu Jogi as UHS exam
a) Cavernous sinus infection
b) Frontal sinusitis 25. A patient is administered
sulphonamides for urinary tract
c) Orbital cellulitis infection. 3 days later he developed flu
d) Petrous part of temporal bone like symptoms, hemorrhagic crusting
e) Sphenoidal sinusitis of lid margins, blistering and
hemorrhagic crusting of lips and
Ans. (D) Ref. Parsons as UHS exam necrotic bullous lesions on skin. What
23. A young man presents with severely is your diagnosis?
chemosed conjunctiva and pulsatile a) Atopic keratoconjunctivitis
proptosis in his left eye. There is bruit
b) Ligneous conjunctivitis
and thrill in the eye. He is having slight
ptosis also in the same eye. He gives c) Mucous membrane pemphigoid
history of head injury in a road traffic d) Stevens Johnson syndrome
accident few weeks back. What is your e) Xerophthalmia
provisional diagnosis?
Ans. (D) as UHS exam
a) Meningoencephalocele
b) Cavernous sinus thrombosis 26. A labourer is brought to the
emergency, who has sustained left eye
c) Carotid cavernous fistula injury while hammering a stone. On
d) Orbital varices examination, his left eye is red. There
e) Orbitalhaemangioma is corneal scar at 3 o'clock position.
There is blood in anterior chamber. He
Ans. (C) has developed traumatic cataract and
24. A 45 year old diabetic patient with a fundus is not visible. What will you do
HbAlc of 14.5%develops a boil on left immediately to exclude any intraocular
side of nose. 2 days later he presents foreign body?
with Pain, Chemosis and Decreased a) Check Intraocular pressure (IOP)

45
b) MRI e) Optic neuritis
c) A-Scan Ans. (B)
d) X-Ray left orbit AP & lateral view 30. A 50 year old research officer in
e) Direct opthalmoscopic examination. Agricultural University visits an
Ans. (D) Ref. Kanski as UHS exam ophthalmologist for a routine
refraction check. His corrected visual
27. The best emergency treatment of an acuities are 6/6 both eyes with
alkali burn at the site of injury is: refractive error of -1 Dioptres both
a) Wash the eye with vinegar to eyes. His intra ocular pressures were
neutralize the alkali recorded as 27mmHg both eyes. There
b) Immediately pad the eye is no relative afferent pupillary defect.
The optic disc are normal with no
c) Give the patient oral antibiotics visual field defect. The most likely
d) Copiously wash the eye with clean diagnosis is:
water for at least 20 to 30 minutes
a) Myopic degeneration of the retina
e) Inject steroids subconjunctivaly
b) Ocular hypertension
Ans. (D) c) Primary open angle glaucoma
28. A 10 years old boy was hit on his right d) Pscudoexfolitive glaucoma
eye with a ball while playing cricket. e) Secondary glaucoma
He is brought to the OPD with the
complaints of diplopia, black and Ans. (B) Ref. Parsons as UHS exam
sunken eye. On examination, his lower 31. A Peshawar University student is hit in
eye lid is swollen. His right eye was the right eye with a tennis ball. Once
sunken. His eye movements was the ecchymosis around the eye has
restricted in up gaze. What can be the settled down he complains of double
possible cause for his diplopia? vision with numbness in the region of
a) Medial orbital wall fracture Intra orbital nerve. There is
b) Superior rectus paralysis enophthalmos of the right eye. The
most likely cause of post traumatic
c) Orbital floor fracture
double vision is:
d) Orbital haematoma
a) Blow out fracture of orbit
c) Temporal orbital wall fracture b) Conical rupture
Ans. (C) Ref. Renu Jogi as UHS exam c) Hyphaema
29. A mechanic presented with history of d) Retinal detachment
trauma while hammering a chisel. His e) Traumatic optic neuropathy
visual acuity was perception of light.
His fundus view was not clear on both Ans. (A) as UHS exam
direct and indirect ophthalmoscopy. 32. A Ironsmith is hammering iron bar
On X-ray orbit he was diagnosed to be with a metal hammer. He notices that
having intraocular foreign body, most something went into his left eye. On
probably an iron one .If his intraocular examination his visual acuities are 6/6
foreign body is not removed, he is both eyes, there is No pupillary defect.
most likely going to develop which of On anterior segment examination
the following complication? there is a small Sub conjunctival
a) Chalcosis Haemorrhage 5 mm from the limbus
b) Siderosis on its temporal aspect. Cornea is clear
and anterior segment is quiet. On
c) Retinal hemorrhage
direct ophthalmoscopy the posterior
d) Vitreous hemorrhage pole of left fundus is normal but there

46
OCULAR TRAUMA UHS Solved MCQ’s Rana
Haseeb

is localised vitreous haemorrhage in c) Cavernous haemangioma


the infero temporal quadrant of the eye d) Thyroid ophthalmopathy
obscuring the-retina underneath. It
e) Lymphomatous deposits
was decided to rule out intra ocular
foreign body. One of the following is Ans. (D) Ref. Renu Jogi as UHS exam
contra indicated in localising 35. A painter of 40 years got victim with
intraocular foreign body in this fresh lime getting into the right eye
patient: while working on a ceiling. He was
a) CT Scan immediately taken to hospital. The
b) Indirect Ophthalmoscopy most effective treatment for this
c) MRI Scan of eye patient would be:
d) Ultrasound B Scan of eye a) Removal of particulate chemical
matter
e) X-Ray Orbits.
b) Irrigation of the eye immediately
Ans. (C) Ref. Kanski as UHS exam c) Antibiotics drops
33. A 30 year old metal worker sustains d) Ascorbate topically
trauma to his right eye with a metal
e) Limbal stem cell transplantation
rod. He is found to have corneal
rupture with iris prolapsed. He is Ans. (B) as UHS exam
operated upon under general 36. Siderosis is due to complication of
anaesthesia, during the surgery the which type of intra ocular foreign
iris is abscised and corneal wound body:
repaired with interrupted sutures, 4
weeks later he presents with pain, a) Copper
photophobia, floaters and decreased b) Iron
vision in the left eye. On examination c) Sodium
the visual acuity is 6/12 with normal
d) Potassium
intra ocular pressure. He has
inflammatory cells in anterior chamber e) Magnesium
and vitreous with choroidal nodules. Ans. (B)
There are deposits of inflammatory
37. The most common cause of
cells on the corneal endothelium but
sympathetic ophthalmitis is:
no corneal staining. The most likely
diagnosis is: a) Chemical burns
a) Conjunctivitis b) Intra ocular foreign body
b) Keratitis c) Perforating injury at the limbus
c) Low tension glaucoma d) Enucleation of sympathising eye
d) Scleritis e) Vitreous loss during cataract surgery
e) Sympathetic ophthalmia Ans. (C) Ref. Parsons as UHS exam
Ans. (E) 38. Chalcosis is an ocular condition which
develops in the eye in case of:
34. A young female of 40 years presents
with axial proptosis for 5 months. CT a) Iron Intra ocular foreign body.
scan advised by the ophthalmologist b) Chemical burn with the lime water.
revealed enlargement of extraocular c) Trauma with a piece of glass.
muscles. The most likely diagnosis is:
d) Copper intraocular foreign body.
a) Optic nerve glioma
b) Meningioma

47
e) Trauma to the eye with a piece of a) Elevation, intorsion, abduction
chalk. b) Elevation, intorsion, depression
Ans. (D) c) Elevation, extorsion, adduction
39. The commonest cause of unilateral d) Elevation, extorsion, depression
proptosis in a young lady is: c) Depression, intorsion, abduction
a) Thyrotoxicosis Ans. (B) Ref. Parsons as UHS exam
b) Metastatic deposits
45. A 70 years old lady, who was taking
c) Haemangioma oral steroids for joints pain, presented
d) Orbital cellulitis to eye OPD with Excruciating pain,
e) Trauma watering and photophobia in the left
eye for two days. On examination
Ans. (D) as UHS exam there was lid oedema and
40. In children the common association of maculopapular rash on left upper lid,
orbital cellulitis is with: scalp and tip of the nose (in the
a) Meningitis distribution of 1st division of
Trigeminal Nerve). The Conjunctiva
b) Paranasal sinusitis shows chemosis and there is punctate
c) Measles staying of Cornea. Most probable
d) Septicemia diagnosis is?
e) Retinoblastoma a) Gonococcal Conjunctivitis
Ans. (B) Ref. Renu Jogi as UHS exam b) Herpes simplex keratitis
c) Herpes Zoster Ophthalmicus (H2O)
41. Swelling behind the ear is diagnostic of
d) Orbital cellulitis
a) Cavernous sinus thrombosis
e) Preseptal cellulitis
b) Orbital cellulitis
c) Unilateral proptosis Ans. (C)
d) Bilateral proptosis 46. In the primary position, the primary
action of the superior rectus muscle is
Ans. (A) as UHS exam
a) Depression
42. The most dangerous complication of
b) Adduction
orbital cellulitis is
c) Elevation
a) Abscess formation
d) Intorsion
b) Proptosis
c) Diplopia Ans. (C) as UHS exam
d) Cerebral involvement 47. The vertical recti form an angle with
the optical axis
Ans. (D) Ref. Renu Jogi as UHS exam
a) 45°
43. When the eye is medially rotated, the
b) 23°
prime depressor muscle of eye ball:
c) 51°
a) Inferior rectus
d) 67°
b) Inferior oblique
c) Inferior rectus and inferior oblique Ans. (B) as UHS exam
d) Lateral rectus 48. There are following cardinal positions
of gaze
e) Superior oblique
a) 9
Ans. (E) Ref. Renu Jogi as UHS exam
b) 6
44. The action of superior rectus is:

48
OCULAR TRAUMA UHS Solved MCQ’s Rana
Haseeb

c) 7
d) 8
Ans. (B) as UHS exam
49. Action of right superior oblique is
a) Laevodepression
b) Laevoelevation
c) Dextrodepression
d) Dextroelevation
Ans. (A) as UHS exam
50. An 18 year old boy comes to the eye
causality with history of injury with a
tennis ball. On examination there is no
perforation but there is hyphaema. The
most likely source of the blood is:
a) Iris vessels
b) Major Arterial Circle of iris
c) Minor Arterial Circle of Iris
d) Short posterior ciliary vessels
e) Long posterior ciliary vessels
Ans. (B)
51. A young child of 9 years presents to
the ophthalmologist with fever for the
last two weeks. The left eye lid is
oedematous and tender. The infection
is seem to be anterior to orbital
septum. The most likely diagnosis is:
a) Orbital cellulitis
b) Preseptal cellulitis
c) Panaophthalmitis
d) Conjunctivitis
e) Insect bite
Ans. (B) Ref. Kanski as UHS exam

49
TOPIC #

SQUINT / ERRORS OF
10 REFRACTION
1. Accommodation is maximum in a) Object moves against the movement
a) Childhood of the lens, when seen through it
b) Adulthood b) It minimizes the size of the object
c) Middle-age c) Object moves with the movement
when seen through it
d) Old age
d) It diverges the parallel rays of light
Ans. (A) as UHS exam passing through it
2. Esotropia is: e) Rays of light passing through it
a) Alternate squint cannot be brought to focus at one
b) Convergent squint point
c) Divergent squint Ans. (A) as UHS exam
d) Latent squint 6. Incident parallel rays come to a focus
e) None of above posterior to the light sensitive layer of
Ans. (B) as UHS exam retina in

3. Pseudopapillitis is seen in a) Aphakia


a) Hypermetropia b) Hypermetropia
b) Myopia c) Both of the above
c) Presbyopia d) None of the above
d) None of the above Ans. (C)
Ans. (A) Ref. Kanski as UHS exam 7. In compound astigmatism, the parallel
4. WHO defines blindness as: rays of light when pass through
a) Best corrected visual acuity of 3/60 in different meridians of the eye are
the better eye brought to focus as two different lines,
b) Visual field of less than 30 degree in which are:
the better eye a) Both are focused on retina
c) Visual acuity of less than 6/60 in both b) One is focused in front of retina and
eyes
other focused behind the retina
d) Visual fields of less than 30 degrees
in both eyes. c) One is focused on retina and the
other in front of retina
e) Visual fields of 10 degrees in both
eyes d) One is focused on retina and the
other behind the retina
Ans. (A)
e) Both are focused either in front or
5. One of the point of identification of a
behind the retina
convex spherical lens is that:
Ans. (E) Ref. Parsons as UHS exam

50
SQUINT/ERRORS OF REFRACTION UHS Solved MCQ’s Rana
Haseeb

8. In Addition to high IOP and high c) Cataract


vertical cup-disc ratio, risk factors for d) Closed angle glaucoma
POAG include all of the following
except: Ans. (D) as UHS exam
a) Black race 13. A 16 year old female child from tirah
b) Family history has visual acuities of 6/6 right and 6/24
Left. There is no Relative Afferent
c) Hypermetropia Pupillary defect. Her Refractive errors
d) Increasing age are -1.0 Diopter in each eye. She had
e) Myopia exotropia in the left eye since the age
of 1 years but the rest of the
Ans. (C) Ref. Kanski as UHS exam examination of the eyes is normal. The
9. Cylindrical lenses are used in the cause of decreased vision in the left
treatment of: eye is:
a) Aphakia a) Anisometropic Amblyopia
b) Astigmatism b) Exophoria
c) Hypermetropia c) Microtropia
d) Myopia d) Stimulus deprivation Amblyopia
e) Preshyopia e) Strabismic Amblyopia
Ans. (B) as UHS exam Ans. (E) Ref. Kanski as UHS exam
10. In Addition to High IOP and high 14. A 6 month old female child is
vertical cup-disc ratio, risk factors for diagnosed with right cataract. The left
POAG include all of the following eye right eye are normal. The
except ophthalmologist insists onocarrying
a) Black race out urgent right phacoemulsification
with Intra ocular implant to prevent:
b) Family history
a) Anisometropic Amblyopia
c) Hypermetropia
b) Esotropia
d) Increasing age
c) Microtropia
e) Myopia
d) Stimulus deprivation Amblyopia
Ans. (C) Ref. Kanski as UHS exam
e) Strabismic Amblyopia
11. In myopia, which one is true:
Ans. (D)
a) Corneal radius of curvature is less
15. The magnification obtained with a
b) Image forms in front of the retina direct ophthalmoscope is:
when the accommodation is relaxed
a) 5 times
c) Lens is less spherical
b) 10 times
d) Length of eye ball is short
c) 15 times
e) Patient can see far objects clearly
when he exerts accommodation d) 20 times
e) 25 times
Ans. (A)
Ans. (C) as UHS exam
12. The complications of myopia include
all EXCEPT 16. In myopia, which one is true:
a) Vitreous degeneration a) Corneal radius of curvature is less
b) Retinal detachment

51
b) Image forms in front of the retina e) Radial Keratotomy
when the patient accommodates
Ans. (D) Ref. Renu Jogi as UHS exam
c) Lens is less spherical
21. Simple myopic astigmatism means:
d) Length of eye ball is short
a) One meridian is myopic and the
e) Patient can sec far objects clearly other is emmetropic
when he exerts accommodation
b) One meridian is hypermetropic and
Ans. (A) the other emmetropic
17. Best vision in moderate myopia is c) Both meridian are myopic
achieved by: d) Both meridian are hypermetropic
a) Glasses e) Both meridian are emmetropic
b) Laser vision correction procedures
Ans. (B) Ref. Parsons as UHS exam
c) Radial keratotomy
22. Radial keratotomy is useful in
d) Rigid gas permeable lenses
a) Myopia
e) Soft contact lenses
b) Hypermetropia
Ans. (B) Ref. Kanski as UHS exam c) Presbyopia
18. Unilateral aphakia is likely to be d) Aphakia
corrected by any of the following
except: Ans. (A) as UHS exam
a) Anterior chamber intraocular lens 23. Accommodative squint if not treated in
b) Contact lens early childhood can lead to:
c) Epikeratophakia a) Stricture of extra ocular muscles
d) Glasses b) Pthysis bulbi
e) Posterior chamber intraocular lens c) Convergent squint
d) Diplopia
Ans. (D) Ref. Parsons as UHS exam
e) Hypertrophy of ciliary muscles
19. Retinoscopy is done for Examination
of: Ans. (C)
a) Axial length of eye 24. On school admission visual screening
b) Optic nerve (at the age of 5 years) a boy with his
left eye can only read 6/18 on snellens
c) Retina visual acuity chart. The vision does
d) Refractive power of eye not improve on pinhole examination.
e) The fundus On distant direct ophthalmoscopy
there is no media opacity. His
Ans. (D) Ref. Renu Jogi as UHS exam refractive error is +1 diopter sphere in
20. A young lady presented with a right eye and +4 in the left eye. Ocular
refractive error of -4.0 D in her both movements are normal. What is the
eyes. She also gives history of allergic most probable cause of reduced visual
conjunctivitis. She is a working acuity in left eye?
women and does not want to wear a) Anisometropic Amblyopia
glasses. What will be the most suitable
b) Hypertropia
treatment option:
c) Microtropia
a) Soft Contact Lenses
d) Stimulus deprivation Amblyopia
b) Hard Contact Lenses
e) Strabismic Amblyopia
c) PRK
d) Lasik Ans. (A) Ref. Renu Jogi as UHS exam

52
SQUINT/ERRORS OF REFRACTION UHS Solved MCQ’s Rana
Haseeb

25. A newborn is invariably one of the following is not the likely


a) Hypermetropic diagnosis:
b) Myopic a) Vitreous Haemorrhage
c) Astigmatic b) Refractive Error of less than 6
Dioptres
d) Aphakic
c) Macular Degeneration d.
Ans. (C) as UHS exam
d) Central Retinal Artery Occlusion
26. A mother brings her 3 years old child e) Central Retinal Vein Occlusion
with deviation of eyes since birth. The
deviation remains the same in all Ans. (B)
directions of the gaze. The doctor 30. A 6 month old female child is
labelled the case as a non-paralytic diagnosed with right cataract. The Left
squint. The most common cause of eye is normal. The rest of the
such type of squint is: examination of the right eye is normal.
a) Refractive error The Ophthalmologist insists on
b) Abnormal ratio of AC/A carrying out urgent right
phacoemulsification with intra ocular
c) Convergence centre's hypo or hyper
implant to prevent:
activity
a) Anisometropic amblyopia
d) Divergence centre's hypo or hyper
activity b) Esotropia
e) Sensory interference such as c) Microtropia
corneal opacity or congenital d) Stimulus deprivation amblyopia
cataract e) Strabismic amblyopia
Ans. (A) as UHS exam Ans. (D) Ref. Renu Jogi as UHS exam
27. Diopteric power of cornea is: 31. In retinoscopy using a plane mirror,
a) 18 diopters when the mirror is tilted to the right the
b) 23 diopters shadow in the pupil moves to the left
in
c) 33 diopters
a) Hypermetropia
d) 43 diopters
b) Myopia more than –1 D
e) 60 diopters
c) Emmetropia
Ans. (D)
d) Myopia less than –1 D
28. In case of incomitant squint, the angle
Ans. (B) as UHS exam
of deviation of squint is:
a) Always more than 45 degrees 32. Blurring of vision for near work occurs
in
b) Is not equal in all directions of gaze
a) Hypermetropia
c) Vertical in all directions of gaze
b) Presbyopia
d) Is less than 10 degrees.
c) Both of the above
e) Becomes more behind the cover
d) None of the above
Ans. (B) as UHS exam
Ans. (C)
29. In a patient with unilateral sudden
decrease in vision if the visual acuity 33. Out of the following which is the
does not improve with pinhole then shortest acting mydriatic
a) Tropicamide

53
b) Homatropine d) Do his cycloplegic retinoscopy and
c) Cyclopentolate examination under anaesthesia
d) Atropine e) Advise the parents to occlude his left
eye so that he could use his right eye
Ans. (A) as UHS exam
Ans. (E) Ref. Renu Jogi as UHS exam
34. An ophthalmologist was explaining
the causes of refractive errors to the 37. Regarding human eye:
residents. Using your knowledge what a) All refractive errors are corrected to
is the most important cause of long 6/6 with a pin hole
sightedness? b) Astigmatism is corrected with plus
a) Decrease in the axial length of an lens in the prescription
eyeball c) In an uncorrected hypermctropia the
b) Decrease in the curvature of cornea image falls behind the retina
eg. plano cornea d) In myopia (uncorrected) the image
c) Change in refractive index of the lens falls behind the retina
eg. cortical cataract e) Minus lens is used to correct
d) Dislocation of the lens in the vitreous astigmatism
e) Absence of the lens from an eye- Ans. (C)
aphakia
38. Latent hypermetropia is detected
Ans. (A) when following mydriatic is used
35. A 20 years female is interested in a) Adrenaline
getting rid of her glasses. Her b) Phenylephrine
refraction reveals VA of 6/6 with 3.00
c) Cyclopentolate
DS/0.50*900 in right eye and 6/6 in left
eye with 3.50 DS/0.75*900. The best d) Atropine
treatment advice for her would be: Ans. (D) as UHS exam
a) Contact lenses 39. Diplopia is a characteristic feature of
b) Radial keratectomy a) Uniocular concomitant squint
c) Photorefractive keratectomy b) Alternating concomitant squint
d) Laser in situ keratomileusis c) Paralytic squint
e) Removal of clear lens d) Apparent squint
Ans. (D) Ref. Renu Jogi as UHS Ans. (C) as UHS exam
exam
40. Hirschberg test is used to detect
36. A three years old child is brought to
the OPD by his parents with the a) Squint
complaint of deviation of eyes. On b) Field defect
examination, he seems to be having c) Glaucoma
right convergent squint. He is not d) Optic atrophy
cooperative to be examined with the
ophthalmoscope. How will you Ans. (A) as UHS exam
proceed to manage this boy? 41. Hess screen is a record of
a) Prescribe some vitamins and call him a) Primary and secondary deviation
when grows old enough to cooperate
b) Heterophoria
b) Do his forced duction test
c) Fusion
c) Admit him for surgery for his right
convergent squint d) Retinal correspondence

54
SQUINT/ERRORS OF REFRACTION UHS Solved MCQ’s Rana
Haseeb

Ans. (A) as UHS exam 47. Concomitant squint has a better


prognosis if the onset is
42. In concomitant squint
a) The centre and afferent pathways a) Very early in life
are intact b) Childhood
b) The efferent pathway is intact c) At birth
c) The efferent pathway is defective
d) Late in life
d) None of the above
Ans. (D) as UHS exam
Ans. (B) as UHS exam
48. Anisophoria is the condition in which
43. Different grades of binocular vision the deviation of the eyeball is
include all EXCEPT
a) Upwards
a) Stereopsis
b) Outwards
b) Simultaneous macular perception c) Downwards
c) Divergence d) Variable according to direction of
d) Fusion gaze
Ans. (D) as UHS exam Ans. (D) as UHS exam
44. Accomodative squint if not treated in 49. In paralytic squint
early childhood can Ieed to:
a) primary deviation > secondary
a) Stricture of extra ocular muscles. deviation
b) Pthysis bulbi b) Primary deviation < secondary
c) Amblyopia deviation
d) Diplopia c) Primary deviation = secondary
e) Hypertrophy of ciliary muscles deviation
d) None of the above
Ans. (C) Ref. Kanski as UHS exam
Ans. (B) as UHS exam
45. In Weber’s syndrome there is a
a) 3rd cranial nerve palsy 50. Ptosis is typically caused in the
paralysis of
b) 4th cranial nerve palsy
a) 3rd nerve
c) 5th cranial nerve palsy
b) 4th nerve
d) 7th cranial nerve palsy
c) 6th nerve
Ans. (A) as UHS exam
d) 7th nerve
46. In paralytic squint deviation of the eye
Ans. (A) as UHS exam
is present
a) Upwards
b) Inwards
c) Outwards
d) In different directions of gaze
Ans. (D) as UHS exam

55
TOPIC #

CHOROID / IRIS
11 / VITREOUS
weeks later he presents with pain,
1. “Mutton fat” keratic precipitates are photophobia, floaters and decreased
typically seen in vision in the left eye. On examination
a) Acute iritis the Visual acuity is 6/12 with normal
b) Chronic cyclitis intra ocular pressure. He has
c) Central choroiditis inflammatory cells in anterior chamber
and vitreous with choroidal nodules.
d) Juxtapapillary choroiditis
There are deposits of inflammatory
Ans. (B) as UHS exam cells on the corneal endothelium but
2. The most common cause of vitreous no corneal staining. The most likely
hemorrhage in old age is diagnosis is:
a) CRVO a) Conjunctivitis
b) Diabetes b) Keratitis
c) Hypertension c) Low tension Glaucoma
d) Retinal hole
d) Scleritis
e) Trauma
e) Sympathetic Ophthalmia
Ans. (B) as UHS exam
Ans. (E) as UHS exam
3. D-Shaped pupil is seen in:
a) Iridectomy 6. Koeppe’s nodules are characteristic of
b) Iridodialysis a) Granulomatous uveitis
c) Iridodonesis b) Exudative uveitis
d) Iris proplapse c) Posterior uveitis
e) Iridoplegia
d) None of the above
Ans. (B) Ref. Parsons as UHS exam
Ans. (A) Ref. Parsons as UHS exam
4. The term endophthalmitis means
inflammation of 7. Pupil in acute anterior uveitis is:
a) Internal structures of the eye a) Dilated and irregular with good
b) All the structures of the eye reaction
c) Choroid b) Dilated and regular with poor
d) Retina reaction
Ans. (A) as UHS exam c) Miosed and regular with poor
reaction
5. A 30 year old metal worker sustains
trauma to his right eye with a metal d) Miosed and irregular with poor
rod. He is found to have corneal reaction
rupture with iris prolapsed. He is e) Mid-dilated and oval with poor
operated upon under general reaction
anesthesia, during the surgery the iris Ans. (D) Ref. Renu Jogi as UHS exam
is abscised and corneal wound
repaired with interrupted sutures, 4

56
CHOROID / IRIS / VITEROUS UHS Solved MCQ’s Rana
Haseeb

8. Black spots floating in front of eyes is Ans. (A) Ref. Renu Jogi as UHS
a symptom of exam
a) Panophthalmitis 12. A 32 year male presents with a red eye.
b) Endophthalmitis He was referred to the
c) Iritis ophthalmologist with a suspected
diagnosis of uveitis. The most reliable
d) Choroiditis
sign for diagnosing acute anterior
Ans. (D) as UHS exam uveitis is:
9. A patient presented with gross a) Moderate to severe pain
decrease of vision. On torch b) Mild to moderate deterioration of
examination there was peri-limbal vision
congestion of conjunctiva and c) KPs on endothelium
pupilary miosis. Which is the probable d) Cells in anterior chamber
diagnosis?
e) Flare in anterior chamber
a) Acute congestive glaucoma
Ans. (D)
b) Anterior uveitis
c) Conjunctivitis 13. A 40 years female presents with
blurred vision of right eye for the last
d) Keratitis 10 days. Clinically she has ciliary
e) Scleritis congestion, small keratic precipitates
Ans. (B) Ref. Kanski as UHS exam on the endothelium with numerous
cells in the anterior chamber. The
10. In juxtapapillary choroiditis, the most likely diagnosis is:
lesions are seen in
a) HLA-B27 related anterior uveitis
a) Macular area b) Idiopathic anterior uveitis
b) Pripheral part c) Fuchs heterochramic iridocyclitis
c) Around the disc d) Herpes simplex anterior ureitis
d) All over the fundus e) Tuberculosis iridocyclitis
Ans. (C) as UHS exam Ans. (B) Ref. Parsons as UHS exam
11. A 30 years male presented with 14. What would be the most likely
sudden dimness of vision in his right diagnosis in a patient with uveitis that
eye for 4 days. He gives history of shows bilateral and symmetric hilar
backache for which he is taking adenopathy on chest X-ray and has
NSAIDs. On examination visual acuity raised serum angiotensin converting
in affected eye is 6/18. On slit lamp enzyme level.
examination, there is circumcorneal a) Behcet's Disease
congestion and KPs, flare and +3 cells
in anterior chamber. Right pupil shows b) Sarcoidosis
a posterior synechiae at 10 O'clock. c) Syphilis
Most probable diagnosis is? d) Toxoplasmosis
a) Acute anterior uveitis e) Tuberculosis
b) Adenoviral conjunctivitis Ans. (B) as UHS exam
c) Angle closure glaucoma
15. Acute anterior uveitis in young
d) Trachoma individuals is commonly associated
e) Vernal Catarrh with:
a) Toxoplasmosis

57
b) Ankylosing spondylitis 21. Hyaluronic acid is present in:
c) Allergic conjunctivitis a) Aqueous humour
d) Sarcoidosis b) Cornea
e) Marfan's syndrome c) Lens
Ans. (B) d) Retina
16. The common complication of severe e) Vitreous
posterior uveitis can be: Ans (E)
a) Vitreous loss
22. Aniridia is a congenital defect whereby
b) Rhegmatogenous retinal there is
detachment a) Absence of iris
c) Exudative retinal detachment b) More than one pupil
d) Dislocation of lens c) Pupil is displaced from central
e) Pupillary block glaucoma position
Ans. (C) Ref. Renu Jogi as UHS d) Pear-shaped coloboma of iris
exam Ans. (A) as UHS exam
17. Heerfordt’s disease is seen in 23. A Raised ESR and C reactive Protein
a) Toxoplasmosis can be a finding in all except one of the
b) Sarcoidosis following:
c) Tuberculosis a) Anterior Ischaemic Optic Neuropathy
d) Histoplasmosis b) Central Retinal Artery Occlusion
Ans. (B) as UHS exam c) Central Retinal Vein Occlusion
18. The treatment of intraocular malignant d) Optic Neuritis
melanoma of the uveal tract includes e) Vitreous Haemorrhage secondary to
a) Enucleation Posterior Vitreous Detachment
b) Evisceration Ans. (E) Ref. Kanski as UHS exam
c) Exenteration
24. Typical coloboma of the iris is situated
d) Chemotherapy
a) Superiorly
Ans. (A)
b) Inferiorly
19. Which of the following regarding c) Superonasally
atropine is true: d) Inferonasally
a) Causes Anhydrosis
b) Causes Cycloplegia Ans. (D) Ref. Renu Jogi as UHS
exam
c) Causes Hallucinations
d) Causes Hyperthermia 25. A 60 year old Diabetic and
e) All of above Hypertensive man for last 20 years has
developed sudden painless decrease
Ans. (E)
in vision in the right eye. his visual
20. The earliest feature of anterior uveitis acuities are HM and 6/12 Right and left
includes eyes respectively. He has no RAPD. All
a) Keratic precipitate except one can he the cause of
b) Hypopyon reduced vision:
a) Anterior Ischaemic Optic Neuropathy
c) Posterior synechiae
b) Central Retinal Artery Occlusion
d) Aqueous flare
c) Central Retinal Vein Occlusion
Ans. (D) as UHS exam d) Cilio Retinal artery Occlusion

58
CHOROID / IRIS / VITEROUS UHS Solved MCQ’s Rana
Haseeb

e) Vitreous Haemorrhage secondary to d) Central choroidal atrophy


Posterior Vitreous Detachment Ans. (D) as UHS exam
Ans. (E) 30. A 25 year old male gives history of
26. Drug of choice for acute iridocyclitis is sudden painless loss of vision in one
a) Acetazolamide eye for the past 2 weeks is no history
b) Atropine of trauma. On examination the anterior
segment is normal but there is no
c) Antibiotic
fundal glow. Which one of the
d) Aspirin
following is the most likely cause?
Ans. (B) as UHS exam a) Acute attack of angle closure
27. In a patient with Sudden Decrease in glaucoma
vision and a Relative Afferent Pupillary b) Central Retinal V. Occlusion
Defect in the affected eye which of the c) Developmental cataract
following is not the likely diagnosis: d) Optic atrophy
a) Anterior Ischaemic Optic Neuropathy e) Vitreous haemorrhage
b) Central Retinal Artery Occlusion Ans. (E) Ref. Renu Jogi as UHS exam
c) Central Retinal Vein Occlusion 31. All of the following are involved in
d) Retrobulbar Optic Neuritis endophthalmitis except:
e) Vitreous Haemorrhage secondary to a) E Lens
Posterior Vitreous Detachment b) Sclera
Ans. (E) c) Retina
d) Uvea
28. In subacute uveitis with glaucoma,
which drug should not be given e) Vitreous
a) Pilocarpine Ans. (B) as UHS exam
b) Timolol maleate 32. A fixed dilated pupil which does not
c) Atropine react to direct or consensual light
d) Serine stimulus could be caused by:
Ans. (A) as UHS exam a) Optic nerve avulsion
29. The common causes of choroid b) Optic Neuritis
detachment include all EXCEPT c) Optic tract Lesion
a) Leaking section postoperatively d) Topical application of atropine
b) Plastic iridocyclitis e) Papi Alloedema
c) New growth Ans. (D)

59

You might also like